HomeDVM3rd Year DVMVeterinary Medicine MCQs With Answers PDF Free Download

Veterinary Medicine MCQs With Answers PDF Free Download [Direct Link]

Welcome to our MCQs Assessment series! In this post, we provide the Veterinary Medicine MCQs With Answers PDF Free Download as well as quizzes on several topics to help you evaluate your understanding and mastery of Veterinary Medicine. These questions are designed to cover key concepts and topics within the subject area, serving as an excellent resource for self-assessment and review.

Diseases of Integumentary System

  1. What is the primary function of the integumentary system?
    a) Facilitating digestion
    b) Maintaining bone density
    c) Protecting internal organs and tissues
    d) Regulating blood sugar levels
View Answer
c) Protecting internal organs and tissues

 

2. Which layer of the skin is primarily responsible for sensory perception?
a) Epidermis
b) Dermis
c) Hypodermis
d) Subcutaneous tissue

View Answer
b) Dermis

 

3. What type of glands secrete sebum, an oily and fatty substance?
a) Eccrine glands
b) Apocrine glands
c) Sweat glands
d) Sebaceous glands

View Answer
d) Sebaceous glands

 

4. What is the main symptom of hyperkeratosis?
a) Red, itching skin
b) Blistering of the skin
c) Thickening of the outer layer of the skin
d) Hair loss

View Answer
c) Thickening of the outer layer of the skin

 

5. Calluses typically occur on which part of the body?
a) Scalp
b) Fingertips
c) Feet
d) Chest

View Answer
c) Feet

 

6. Which vitamin deficiency can result in follicular hyperkeratosis?
a) Vitamin D
b) Vitamin B
c) Vitamin C
d) Vitamin K

View Answer
b) Vitamin B

 

7. How is dermatitis commonly characterized?
a) Itchy, dry skin
b) Nausea and vomiting
c) Joint pain
d) Blurred vision

View Answer
a) Itchy, dry skin

 

8. What is the primary cause of atopic dermatitis (eczema)?
a) Exposure to sunlight
b) Dry skin
c) Immune system dysfunction
d) Ingestion of spicy foods

View Answer
c) Immune system dysfunction

 

9. Which diagnostic test is commonly used to identify allergic contact dermatitis?
a) Blood test
b) X-ray
c) Patch test
d) Urine analysis

View Answer
c) Patch test

 

10. Which layer of the skin is primarily affected by hyperkeratosis?
a) Dermis
b) Epidermis
c) Subcutaneous tissue
d) Hypodermis

View Answer
b) Epidermis

 

11. Which type of glands are most numerous and cover almost the entire body?
a) Sebaceous glands
b) Eccrine glands
c) Sweat glands
d) Apocrine glands

View Answer
b) Eccrine glands

 

12. What is the primary function of the sweat glands?
a) Secreting sebum
b) Regulating body temperature
c) Producing hair follicles
d) Protecting against UV radiation

View Answer
b) Regulating body temperature

 

13. What is the mnemonic used to remember the layers of the epidermis?
a) Good Boys Sing Loudly
b) Big Elephants Are Scary Creatures
c) British and Spanish Grannies Love Cornflakes
d) Cats Always Sleep Gracefully

View Answer
b) Big Elephants Are Scary Creatures

 

14. Which condition causes very red skin and severe blistering of the skin at birth?
a) Hyperkeratosis
b) Dermatitis
c) Eczema
d) Epidermolytic hyperkeratosis

View Answer
d) Epidermolytic hyperkeratosis

 

15. What is the primary treatment goal for hyperkeratosis?
a) Alleviating symptomatology
b) Increasing keratinization
c) Reducing skin hydration
d) Inducing blistering

View Answer
a) Alleviating symptomatology

 

16. What is the primary characteristic of calluses?
a) Uneven thickness
b) Soft keratin center
c) Presence on the scalp
d) Even thickness

View Answer
d) Even thickness

 

17. What is the main symptom of eczema?
a) Swelling and tenderness
b) Thickening of the skin
c) Red, itching skin
d) Dry, cracked skin

View Answer
c) Red, itching skin

 

18. How is atopic dermatitis (eczema) commonly diagnosed?
a) Blood test
b) Physical examination
c) Biopsy
d) MRI scan

View Answer
b) Physical examination

 

19. What type of dermatitis is commonly associated with exposure to allergens?
a) Atopic dermatitis
b) Contact dermatitis
c) Seborrheic dermatitis
d) Neurodermatitis

View Answer
b) Contact dermatitis

 

20. What is the primary cause of pressure-related hyperkeratosis?
a) Excessive hydration
b) Vitamin deficiency
c) Inflammation or irritation to the skin
d) Lack of exposure to sunlight

View Answer
c) Inflammation or irritation to the skin

 

21. What is the primary treatment recommended for reducing inflammation and healing irritation associated with most types of dermatitis?
a) Antibiotics
b) Oral antihistamines
c) Prescription corticosteroid cream
d) Topical antibacterial ointments

View Answer
c) Prescription corticosteroid cream

 

22. What is the characteristic sign of photosensitization in dairy cows?
a) Increased milk production
b) Swelling of affected areas
c) Loss of appetite
d) Hoof abnormalities

View Answer
b) Swelling of affected areas

 

23. Which type of photosensitization is most commonly seen in livestock?
a) Primary photosensitization
b) Type I photosensitization
c) Type II photosensitization
d) Secondary (type III) photosensitization

View Answer
c) Type II photosensitization

 

24. What is a common characteristic of skin cysts?
a) They are always cancerous
b) They are filled with air
c) They may feel like small peas beneath the skin
d) They are caused by abnormal melanocytes

View Answer
c) They may feel like small peas beneath the skin

 

25. How can a doctor diagnose a cyst or tumor located deep within the body?
a) Through physical examination only
b) By requesting a biopsy
c) By performing an ultrasound
d) By conducting a blood test

View Answer
b) By requesting a biopsy

 

26. What is a potential cause of cysts?
a) Skin cancer
b) Genetic conditions
c) Excessive exposure to sunlight
d) Bacterial infections

View Answer
b) Genetic conditions

 

27. Which cells in the epidermis are responsible for producing melanin?
a) Basal cells
b) Squamous cells
c) Melanocytes
d) Keratinocytes

View Answer
c) Melanocytes

 

28. What treatment option involves removing the entire cyst wall to prevent recurrence?
a) Draining the cyst
b) Injecting medicine into the cyst
c) Minor surgery
d) Laser removal

View Answer
c) Minor surgery

 

29. What method involves scraping away tumor tissues with a sharp surgical instrument?
a) Surgical excision
b) Cryosurgery
c) Curettage and electrodesiccation
d) Injection of medication

View Answer
c) Curettage and electrodesiccation

 

30. Which type of skin tumor may not require any treatment?
a) Basal cell carcinoma
b) Melanoma
c) Squamous cell carcinoma
d) Sebaceous adenoma

View Answer
d) Sebaceous adenoma

 

31. What is the primary symptom of cysts located deep within the body?
a) Skin discoloration
b) Pain
c) Itching
d) Swelling

View Answer
b) Pain

 

32. What is the main cause of photosensitivity in livestock?
a) Exposure to chemicals
b) Abnormal hepatic metabolism
c) Genetic predisposition
d) Bacterial infections

View Answer
a) Exposure to chemicals

 

33. Which type of photosensitivity reaction results in blistering and necrosis of the skin?
a) Type I
b) Type II
c) Type III
d) Primary

View Answer
c) Type III

 

34. What type of cells are located in the lower part of the epidermis and produce melanin?
a) Basal cells
b) Squamous cells
c) Melanocytes
d) Keratinocytes

View Answer
c) Melanocytes

 

35. What treatment option involves freezing the tissue with liquid nitrogen?
a) Draining the cyst
b) Laser removal
c) Cryosurgery
d) Minor surgery

View Answer
c) Cryosurgery

 

36. What is the primary treatment for reducing inflammation associated with dermatitis?
a) Antibiotics
b) Oral antihistamines
c) Corticosteroid cream
d) Topical antibacterial ointments

View Answer
c) Corticosteroid cream

 

37. Which method involves cutting into the skin to remove a skin tumor?
a) Cryosurgery
b) Curettage and electrodesiccation
c) Surgical excision
d) Injection of medication

View Answer
c) Surgical excision

 

38. What is a characteristic sign of photosensitivity in dairy cows according to the text?
a) Increased milk production
b) Swelling of affected areas
c) Loss of appetite
d) Hoof abnormalities

View Answer
b) Swelling of affected areas

 

39. What is a common characteristic of skin tumors?
a) They are always benign.
b) They may not require any treatment.
c) They are filled with air.
d) They are caused by abnormal melanocytes

View Answer
b) They may not require any treatment

 

40. Which cells in the epidermis produce new skin cells?
a) Basal cells
b) Squamous cells
c) Melanocytes
d) Keratinocytes

View Answer
a) Basal cells

Download Link

Download PDF

Diseases Of the Alimentary tract

  1. What are the primary functions of the alimentary tract?
    a) Prehension and excretion
    b) Digestion and absorption of food and water
    c) Respiration and circulation
    d) Protection and sensation
View Answer
b) Digestion and absorption of food and water

 

2. Which of the following is not a major mode of alimentary dysfunction?
a) Motility
b) Secretion
c) Respiration
d) Absorption

View Answer
c) Respiration

 

3. What is stomatitis?
a) Inflammation of the lungs
b) Inflammation of the mouth
c) Inflammation of the stomach
d) Inflammation of the liver

View Answer
b) Inflammation of the mouth

 

4. What is a common cause of stomatitis?
a) Skin disease
b) Respiratory infection
c) Muscle strain
d) Bone fracture

View Answer
b) Respiratory infection

 

5. Which of the following is a symptom of stomatitis?
a) Runny nose
b) Swollen ankles
c) Mouth ulcers with white or yellow layer
d) Hair loss

View Answer
c) Mouth ulcers with white or yellow layer

 

6. What is pharyngitis?
a) Inflammation of the stomach
b) Inflammation of the throat
c) Inflammation of the liver
d) Inflammation of the kidneys

View Answer
b) Inflammation of the throat

 

7. What is the main symptom of viral pharyngitis?
a) Red eyes
b) Rash on the skin
c) Sore throat
d) Severe headache

View Answer
c) Sore throat

 

8. Which treatment is necessary for bacterial pharyngitis?
a) Antibiotics
b) Antihistamines
c) Painkillers
d) Steroids

View Answer
Antibiotics

 

9. What is obstruction in the context of alimentary tract dysfunction?
a) Enlargement of organs
b) Narrowing of blood vessels
c) Blockage of a passageway
d) Inflammation of tissues

View Answer
Blockage of a passageway

 

10. What are the signs and symptoms of intestinal obstruction?
a) Dizziness and fatigue
b) Abdominal pain and constipation
c) Headache and fever
d) Nausea and vomiting

View Answer
b) Abdominal pain and constipation

 

11. Which of the following is a mechanical cause of intestinal obstruction?
a) Viral infection
b) Gallstones
c) Allergy
d) Skin disease

View Answer
b) Gallstones

 

12. What diagnostic test may confirm a diagnosis of intestinal obstruction?
a) Blood test
b) Urine test
c) X-ray
d) MRI scan

View Answer
c) X-ray

 

13. What is peritonitis?
a) Inflammation of the peritoneum
b) Inflammation of the brain
c) Inflammation of the heart
d) Inflammation of the lungs

View Answer
a) Inflammation of the peritoneum

 

14. What are the first symptoms of peritonitis?
a) Headache and fatigue
b) Chest pain and shortness of breath
c) Abdominal pain and nausea
d) Joint pain and swelling

View Answer
c) Abdominal pain and nausea

 

15. How is peritonitis treated?
a) Intravenous antibiotics
b) Oral medication
c) Physical therapy
d) Surgery

View Answer
d) Surgery

 

16. What is ruminal acidosis?
a) A metabolic disorder of gastrointestinal origin in ruminants
b) A neurological disorder in humans
c) A respiratory disorder in birds
d) A cardiovascular disorder in fish

View Answer
a) A metabolic disorder of gastrointestinal origin in ruminants

 

17. What triggers systemic acidosis in acute lactic acidosis?
a) Prolonged exposure to high ruminal pH
b) Prolonged exposure to low ruminal pH
c) Lack of water intake
d) Overconsumption of hay

View Answer
b) Prolonged exposure to low ruminal pH

 

18. What is the normal pH range of the rumen?
a) 5.5 to 6.0
b) 6.5 to 7.0
c) 7.5 to 8.0
d) 8.5 to 9.0

View Answer
b) 6.5 to 7.0

 

19. What effect does ruminal acidosis have on the appetite and production of cattle?
a) Stimulates appetite and increases production
b) Depresses appetite and production
c) Has no effect on appetite and production
d) Reduces appetite but increases production

View Answer
b) Depresses appetite and production

 

20. Which of the following statements regarding ruminal acidosis is true?
a) It affects primarily carnivores
b) It is caused by overconsumption of fiber-rich diets
c) It leads to an increase in rumen pH
d) It is classified based on clinical manifestations and biochemical changes

View Answer
d) It is classified based on clinical manifestations and biochemical changes

 

21. What is the primary cause of ruminal acidosis in cattle?
A) High level of rapidly digestible carbohydrate
B) Excessive intake of protein-rich feed
C) Low dietary fiber content
D) Lack of essential minerals in the diet

View Answer
A) High level of rapidly digestible carbohydrate

 

22. Which of the following symptoms is NOT associated with acute ruminal acidosis in cattle?
A) Depression
B) Elevated heart rate
C) Diarrhea
D) Increased feed intake

View Answer
D) Increased feed intake

 

23. What is the primary treatment for ruminal tympany (bloat) in ruminant animals?
A) Removal of gases through trocar or cannula
B) Intravenous antibiotics
C) Oral administration of antihistamines
D) Restricted access to water

View Answer
A) Removal of gases through trocar or cannula

 

24. Which of the following is a characteristic symptom of enteritis?
A) Elevated body temperature
B) Decreased respiratory rate
C) Increased appetite
D) Improved digestion

View Answer
A) Elevated body temperature

 

25. Which type of enteritis is commonly caused by ingesting contaminated food or water?
A) Medication-induced enteritis
B) Radiation-induced enteritis
C) Alcohol-induced enteritis
D) Infectious enteritis

View Answer
D) Infectious enteritis

 

26. What is the most common bacterial cause of infectious enteritis?
A) Salmonella
B) Escherichia coli
C) Campylobacter jejuni
D) Yersinia enterocolitica

View Answer
A) Salmonella

 

27. Which of the following is NOT a common symptom of sub-acute ruminal acidosis in cattle?
A) Reduced feed intake
B) Poor body condition and weight loss
C) Elevated body temperature
D) Unexplained diarrhea

View Answer
C) Elevated body temperature

 

28. What is the primary source of excessive gas accumulation in ruminal tympany?
A) Consumption of high-fiber feed
B) Accumulation of fine particles in the rumen
C) Inadequate water intake
D) Overconsumption of protein-rich feed

View Answer
B) Accumulation of fine particles in the rumen

 

29. What can be used as an alternative to antifoaming agents for treating ruminal tympany?
A) Intravenous antibiotics
B) Probiotics
C) Vegetable oil
D) Digestive enzymes

View Answer
C) Vegetable oil

 

30. In which season is ruminal alkalosis commonly observed in cattle?
A) Spring
B) Summer
C) Fall
D) Winter

View Answer
B) Summer

 

31. What is the primary treatment for acute ruminal acidosis in cattle?
A) Sodium bicarbonate
B) Intravenous fluids
C) Oral antibiotics
D) Rest and isolation

View Answer
B) Intravenous fluids

 

32. Which type of ruminal bloat is characterized by the accumulation of frothy gases?
A) Free-gas bloat
B) Primary bloat
C) Secondary bloat
D) Tympany

View Answer
C) Secondary bloat

 

33. What is the most common cause of primary ruminal tympany?
A) Excessive water consumption
B) Inadequate fiber intake
C) Accumulation of frothy gases
D) Consumption of legumes/grains

View Answer
C) Accumulation of frothy gases

 

34. Which of the following is NOT a potential cause of enteritis?
A) Poor blood flow to the intestine
B) Consumption of contaminated food or water
C) Allergic reaction to medication
D) Inflammatory conditions such as Crohn’s disease

View Answer
C) Allergic reaction to medication

 

35. Which type of acidosis is commonly seen in dairy cattle as a result of feeding increased concentrates compared to forage?
A) Sub-acute ruminal acidosis
B) Acute ruminal acidosis
C) Metabolic acidosis
D) Ruminal alkalosis

View Answer
A) Sub-acute ruminal acidosis

Download Link

Download PDF

Diseases Of Respiratory System

  1. What is the primary function of the respiratory system?
    a) Blood circulation
    b) Digestion
    c) Gas exchange
    d) Muscle movement
View Answer
c) Gas exchange

 

2. Which of the following is NOT a part of the respiratory system?
a) Lungs
b) Heart
c) Nose
d) Throat

View Answer
b) Heart

 

3. What is the medical term for inflammation of the mucous membrane of the nose?
a) Rhinoplasty
b) Rhinovirus
c) Rhinitis
d) Rhizome

View Answer
c) Rhinitis

 

4. Which of the following is a symptom of allergic rhinitis?
a) Dry cough
b) Watery eyes
c) Sore throat
d) Chest pain

View Answer
b) Watery eyes

 

5. What is the most common cause of laryngitis?
a) Bacterial infection
b) Excessive alcohol intake
c) Allergic reaction
d) Viral infection

View Answer
d) Viral infection

 

6. What diagnostic technique involves visually examining the vocal cords using a light and mirror?
a) Bronchoscopy
b) Laryngoscopy
c) Endoscopy
d) Rhinoscopy

View Answer
b) Laryngoscopy

 

7. Pulmonary congestion primarily affects which region of the lungs first?
a) Alveoli
b) Interstitial space
c) Hilar region
d) Bronchioles

View Answer
b) Interstitial space

 

8. Which condition is characterized by excess fluid in the lungs, making breathing difficult?
a) Pulmonary congestion
b) Pneumothorax
c) Pulmonary embolism
d) Pulmonary edema

View Answer
d) Pulmonary edema

 

9. What symptom is commonly associated with pulmonary edema?
a) High fever
b) Chest pain
c) Cold, clammy skin
d) Muscle weakness

View Answer
b) Chest pain

 

10. Which of the following is NOT a cause of pulmonary congestion and edema?
a) Near drowning
b) Adverse drug reaction
c) Chronic obstructive pulmonary disease (COPD)
d) Exposure to certain toxins

View Answer
c) Chronic obstructive pulmonary disease (COPD)

 

11. Which of the following is a function of the respiratory system?
a) Digestion
b) Sound production
c) Blood circulation
d) Muscle coordination

View Answer
b) Sound production

 

12. What is the role of histamine in allergic rhinitis?
a) It helps defend against allergens
b) It regulates blood pressure
c) It aids in digestion
d) It promotes muscle relaxation

View Answer
a) It helps defend against allergens

 

13. What symptom distinguishes allergic rhinitis from a common cold?
a) Fever
b) Sneezing
c) Coughing
d) Headache

View Answer
a) Fever

 

14. Chronic laryngitis is typically caused by which of the following?
a) Bacterial infection
b) Acid reflux
c) Allergic reaction
d) Viral infection

View Answer
b) Acid reflux

 

15. Which diagnostic test allows for visual examination of the larynx using a tiny mirror?
a) Rhinoscopy
b) Bronchoscopy
c) Laryngoscopy
d) Endoscopy

View Answer
c) Laryngoscopy

 

16. What is the primary treatment for acute laryngitis?
a) Antibiotics
b) Corticosteroids
c) Voice rest and hydration
d) Surgery

View Answer
c) Voice rest and hydration

 

17. Which condition is characterized by impaired gas exchange and arterial hypoxemia?
a) Pulmonary congestion
b) Pulmonary edema
c) Bronchitis
d) Pneumonia

View Answer
d) Pneumonia

 

18. What is the primary cause of pulmonary edema?
a) Heart problems
b) Lung cancer
c) Bacterial infection
d) Exposure to toxins

View Answer
a) Heart problems

 

19. What symptom is commonly associated with acute bronchitis?
a) Fever
b) Sore throat
c) Productive cough
d) Itchy eyes

View Answer
c) Productive cough

 

20. Which part of the respiratory system is responsible for sound production?
a) Trachea
b) Lungs
c) Voice box (larynx)
d) Bronchi

View Answer
c) Voice box (larynx)

 

21. What is the role of corticosteroids in the treatment of laryngitis?
a) They reduce vocal cord inflammation
b) They act as pain relievers
c) They inhibit bacterial growth
d) They suppress the immune system

View Answer
a) They reduce vocal cord inflammation

 

22. What is the primary symptom of bacterial tracheitis?
a) Runny nose
b) Wheezing
c) High fever
d) Hoarseness

View Answer
c) High fever

 

23. What is the function of saline nasal sprays in treating rhinitis?
a) They kill bacteria
b) They reduce inflammation
c) They flush irritants from the nose
d) They suppress coughing

View Answer
c) They flush irritants from the nose

 

24. Which of the following is NOT a symptom of rhinitis?
a) Nasal congestion
b) Sneezing
c) Fatigue
d) Diarrhea

View Answer
d) Diarrhea

 

25. What is the primary function of anti-drip anticholinergic nasal sprays in treating rhinitis?
a) They reduce mucus production
b) They relieve pain
c) They dilate blood vessels
d) They suppress coughing

View Answer
a) They reduce mucus production

 

26. What diagnostic method is commonly utilized to confirm the presence of pulmonary edema?
A) Blood tests
B) Pulse oximetry
C) Chest X-ray
D) Sputum culture

View Answer
C) Chest X-ray

 

27. Which medication is typically prescribed to decrease pressure caused by excess fluid in the heart and lungs?
A) Morphine
B) Blood pressure drugs
C) Antibiotics
D) Diuretics

View Answer
D) Diuretics

 

28. What is the primary cause of pneumonia?
A) Exposure to air pollution
B) Smoking
C) Bacterial infections
D) Viral infections

View Answer
D) Viral infections

 

29. Which symptom is NOT typically associated with pneumonia?
A) Cough producing greenish mucus
B) Rapid, shallow breathing
C) Loss of appetite and fatigue
D) Wheezing

View Answer
D) Wheezing

 

30. What is the most common bacterial type that causes pneumonia?
A) Streptococcus pneumoniae
B) Haemophilus influenzae
C) Mycobacterium tuberculosis
D) Legionella pneumophila

View Answer
A) Streptococcus pneumoniae

 

31. What is the primary cause of pulmonary emphysema?
A) Exposure to chemical fumes
B) Fungal infections
C) Genetic factors
D) Bacterial infections

View Answer
C) Genetic factors

 

32. Which of the following is a symptom of pulmonary emphysema?
A) Fever
B) Productive cough
C) Bradycardia
D) Increased appetite

View Answer
B) Productive cough

 

33. How is pulmonary emphysema typically diagnosed?
A) MRI scan
B) Spirometry
C) Electrocardiogram
D) Urinalysis

View Answer
B) Spirometry

 

34. Which treatment option is NOT typically used for pulmonary emphysema?
A) Lung transplant
B) Antibiotics
C) Chemotherapy
D) Oxygen therapy

View Answer
C) Chemotherapy

 

35. What is the primary cause of hemothorax?
A) Lung cancer
B) Blood clotting disorders
C) Tuberculosis
D) Sinusitis

View Answer
B) Blood clotting disorders

 

36. Which diagnostic method may be utilized to diagnose hemothorax?
A) EEG
B) MRI
C) CT scan
D) Ultrasound

View Answer
D) Ultrasound

 

37. What is the main treatment for hemothorax?
A) Antibiotics
B) Radiation therapy
C) Drainage of blood from the chest cavity
D) Chemotherapy

View Answer
C) Drainage of blood from the chest cavity

 

38. What is epistaxis?
A) Accumulation of blood in the pleural cavity
B) Acute hemorrhage from the nostril or nasal cavity
C) Coughing up of blood from the lungs
D) Bleeding from the stomach

View Answer
B) Acute hemorrhage from the nostril or nasal cavity

 

39. Which symptom is NOT associated with epistaxis?
A) Persistent dry nose
B) Stuffy nose and colored drainage
C) Difficulty breathing through the nose
D) Chest pain

View Answer
D) Chest pain

 

40. What is a common cause of nosebleeds?
A) Excessive crying
B) Blowing nose with force
C) Excessive laughter
D) Sneezing

View Answer
B) Blowing nose with force

 

41. What diagnostic tests may be performed if a foreign body or tumor is suspected in epistaxis?
A) CBC
B) CT scan
C) Chest X-ray
D) Echocardiogram

View Answer
B) CT scan

 

42. Which treatment option may be used for epistaxis?
A) Radiation therapy
B) Topical vasoconstriction
C) Cardiac catheterization
D) Chemotherapy

View Answer
B) Topical vasoconstriction

 

43. What is hemoptysis?
A) Acute hemorrhage from the nostril or nasal cavity
B) Accumulation of blood in the pleural cavity
C) Coughing up of blood or bloody mucus from the lungs
D) Bleeding from the stomach

View Answer
C) Coughing up of blood or bloody mucus from the lungs

 

44. Which symptom is typically NOT associated with hemoptysis?
A) Coughing up of blood or bloody sputum
B) Stuffy nose and colored drainage
C) Feeling anxious or nervous
D) Difficulty breathing

View Answer
B) Stuffy nose and colored drainage

 

45. What is a possible consequence of prolonged or recurrent nosebleeds?
A) Cardiac arrhythmia
B) Anemia
C) Respiratory failure
D) Hypernatremia

View Answer
B) Anemia

 

46. What is the primary cause of nosebleeds?
A) Excessive laughter
B) Sneezing
C) Nose picking
D) Blowing nose gently

View Answer
C) Nose picking

 

47. What diagnostic tests may be recommended for epistaxis if a bleeding disorder is suspected?
A) CT scan
B) MRI
C) CBC, PT, and PTT
D) Chest X-ray

View Answer
C) CBC, PT, and PTT

 

48. Which treatment option may be utilized for epistaxis?
A) Oral antibiotics
B) Topical vasoconstriction
C) Chemotherapy
D) Cardiac catheterization

View Answer
B) Topical vasoconstriction

 

49. What is the primary symptom of hemothorax?
A) Persistent dry nose
B) Chest pain and difficulty breathing
C) Coughing up of blood
D) Itchy, watery eyes

View Answer
B) Chest pain and difficulty breathing

 

50. Which treatment method is commonly employed for hemothorax?
A) Oxygen therapy
B) Chemotherapy
C) Drainage of blood from the chest cavity
D) Oral antihistamines

View Answer
C) Drainage of blood from the chest cavity

 

What Is Fever

  1. Fever is primarily characterized by:
    a) Decreased sensitivity to pain
    b) Increased metabolism
    c) Decreased antibody production
    d) Reduced heat production
View Answer
b) Increased metabolism

 

2. Which of the following is not a cause of fever?
a) Bacterial infection
b) Mechanical injuries
c) Surgical operation
d) Hot weather conditions

View Answer
b) Mechanical injuries

 

3. What type of fever occurs due to tissue damage caused by foreign substances?
a) Septic fever
b) Aseptic fever
c) Hyperthermia
d) Toxemia fever

View Answer
b) Aseptic fever

 

4. Symptoms of hyperthermia include all of the following except:
a) Excessive sweating
b) Flushed or red skin
c) Muscle cramps
d) Slow, shallow breathing

View Answer
d) Slow, shallow breathing

 

5. Which of the following is not a symptom of hypothermia?
a) Shivering
b) Clumsiness or lack of coordination
c) Excessive sweating
d) Confusion or memory loss

View Answer
c) Excessive sweating

 

6. What is the primary cause of hypothermia?
a) Exposure to cold weather
b) Exposure to hot weather
c) Intense physical exercise
d) Excessive sweating

View Answer
a) Exposure to cold weather

 

7. What condition is characterized by the presence of toxic substances in the blood?
a) Septicemia
b) Hypersensitivity
c) Hyperthermia
d) Toxemia

View Answer
d) Toxemia

 

8. Symptoms of toxemia may include:
a) Increased urine output
b) Severe headaches
c) Normal protein levels in urine
d) Decreased appetite

View Answer
b) Severe headaches

 

9. Septicemia is also known as:
a) Fever
b) Hypersensitivity
c) Blood poisoning
d) Toxemia

View Answer
c) Blood poisoning

 

10. What is the most common cause of sepsis?
a) Fungal infections
b) Parasitic infections
c) Bacterial infections
d) Viral infections

View Answer
c) Bacterial infections

 

11. Which type of shock is caused by damage to the nervous system?
a) Cardiogenic shock
b) Hypovolemic shock
c) Anaphylactic shock
d) Neurogenic shock

View Answer
d) Neurogenic shock

 

12. Signs and symptoms of shock may include:
a) Warm, dry skin
b) Slow pulse
c) Rapid breathing
d) Enlarged pupils

View Answer
c) Rapid breathing

 

13. What term refers to undesirable reactions produced by the normal immune system?
a) Fever
b) Hypersensitivity
c) Hyperthermia
d) Septicemia

View Answer
b) Hypersensitivity

 

14. Which of the following is not a symptom of acute hypersensitivity pneumonitis?
a) Fever and chills
b) Rales
c) Weight gain
d) Shortness of breath

View Answer
c) Weight gain

 

15. What is the primary treatment for hypothermia?
a) Increasing exposure to cold weather
b) Administering antipyretic drugs
c) Warming the body back to a normal temperature
d) Increasing fluid intake

View Answer
c) Warming the body back to a normal temperature

 

16. What is the characteristic feature of septic fever?
a) Heat production due to metabolism
b) Increase in antibodies production
c) Presence of bacterial, viral, fungal, or protozoal infection
d) Decrease in tissue injuries

View Answer
c) Presence of bacterial, viral, fungal, or protozoal infection

 

17. Which condition is associated with an abnormally high body temperature?
a) Fever
b) Hypothermia
c) Septicemia
d) Hypersensitivity

View Answer
a) Fever

 

18. What is the primary symptom of hypothermia?
a) Excessive sweating
b) Shivering
c) Muscle cramps
d) Flushed skin

View Answer
b) Shivering

 

19. What type of shock is caused by heart problems?
a) Cardiogenic shock
b) Hypovolemic shock
c) Anaphylactic shock
d) Septic shock

View Answer
a) Cardiogenic shock

 

20. What are the symptoms of sepsis?
a) Decreased body temperature
b) Increased urine output
c) Rapid heartbeat
d) Bright red, cold skin

View Answer
c) Rapid heartbeat

 

21. What is the primary cause of septicemia?
a) Fungal infections
b) Bacterial infections
c) Viral infections
d) Parasitic infections

View Answer
b) Bacterial infections

 

22. Which type of shock results from exposure to cold weather?
a) Cardiogenic shock
b) Hypovolemic shock
c) Hypothermia
d) Neurogenic shock

View Answer
c) Hypothermia

 

23. What are the symptoms of hyperthermia?
a) Slow, shallow breathing
b) Clumsiness or lack of coordination
c) Bluish tinge to lips or fingernails
d) Excessive sweating

View Answer
d) Excessive sweating

 

24. What term refers to the presence of toxic substances in the blood?
a) Fever
b) Septicemia
c) Toxemia
d) Hypersensitivity

View Answer
c) Toxemia

 

25. What is the primary treatment for septicemia?
a) Administering antipyretic drugs
b) Increasing fluid intake
c) Treating the underlying bacterial infection
d) Applying cold compresses

View Answer
c) Treating the underlying bacterial infection

 

Anemia

1. What is the primary characteristic of anemia?
a) Decrease in platelet count
b) Low oxygen carrying capacity of blood
c) Increase in white blood cell count
d) Elevated hemoglobin levels
View Answer

b) Low oxygen carrying capacity of blood

 

2. Which of the following is NOT a symptom of anemia?
a) Weakness
b) Fever
c) Fatigue
d) Tachycardia
View Answer

b) Fever

 

3. What are the main causes of anemia?
a) Increased RBC production
b) Excessive hemoglobin concentration
c) Blood loss, hemolysis, decreased RBC production
d) Elevated platelet levels
View Answer

c) Blood loss, hemolysis, decreased RBC production

 

4. Which of the following conditions is NOT associated with anemia?
a) Sickle cell disease
b) Malaria
c) Diabetes
d) Thalassemia
View Answer

c) Diabetes

 

5. How are anemias classified based on morphology?
a) By platelet count
b) By RBC size and hemoglobin concentration
c) By white blood cell count
d) By clotting time
View Answer

b) By RBC size and hemoglobin concentration

 

6. What is the normal range for mean corpuscular volume (MCV)?
a) 10-30 fL
b) 50-80 fL
c) 80-100 fL
d) 120-150 fL
View Answer

c) 80-100 fL

 

7. Which type of anemia is characterized by low MCV and MCHC?
a) Microcytic hypochromic anemia
b) Normocytic normochromic anemia
c) Macrocytic normochromic anemia
d) Hyperchromic anemia
View Answer

a) Microcytic hypochromic anemia

 

8. What condition is associated with macrocytic normochromic anemia?
a) Iron deficiency anemia
b) Pernicious anemia/B12 deficiency anemia
c) Sickle cell anemia
d) Thalassemia
View Answer

b) Pernicious anemia/B12 deficiency anemia

 

9. Which of the following is a symptom of anemia?
a) Hypertension
b) Jaundice
c) Bradycardia
d) Pale skin
View Answer

d) Pale skin

 

10. What is the primary characteristic of normocytic normochromic anemia?
a) Normal RBC size and hemoglobin concentration
b) Low RBC count
c) High MCV and MCHC
d) Hyperchromic RBCs
View Answer

a) Normal RBC size and hemoglobin concentration

 

11. Which of the following is NOT a cause of decreased RBC production?
a) Renal disease
b) Malnutrition
c) Sickle cell disease
d) Hemolysis
View Answer

c) Sickle cell disease

 

12. What is the normal range for mean corpuscular hemoglobin concentration (MCHC)?
a) 10-20 g/dL
b) 20-30 g/dL
c) 30-34 g/dL
d) 40-50 g/dL
View Answer

c) 30-34 g/dL

 

13. Which type of anemia is associated with snake venom?
a) Iron deficiency anemia
b) Thalassemia
c) Hemolytic anemia
d) Pernicious anemia
View Answer

c) Hemolytic anemia

 

14. What is the term for erythrocytes with a high MCV?
a) Normocytic
b) Microcytic
c) Macrocytic
d) Hypochromic
View Answer

c) Macrocytic

 

15. What condition results in increased MCHC?
a) Iron deficiency anemia
b) Thalassemia
c) Hemolytic anemia
d) Sickle cell anemia
View Answer

d) Sickle cell anemia

 

16. What type of anemia is associated with decreased MCV and MCHC?
a) Normocytic normochromic anemia
b) Microcytic hypochromic anemia
c) Macrocytic normochromic anemia
d) Hyperchromic anemia
View Answer

b) Microcytic hypochromic anemia

 

17. What is the primary cause of normocytic normochromic anemia?
a) Blood loss
b) Hemolysis
c) Decreased RBC production
d) Vitamin B12 deficiency
View Answer

a) Blood loss

 

18. Which type of anemia is characterized by abnormal RBC size and normal hemoglobin concentration?
a) Microcytic hypochromic anemia
b) Normocytic normochromic anemia
c) Macrocytic normochromic anemia
d) Hyperchromic anemia
View Answer

b) Normocytic normochromic anemia

 

19. What is the main characteristic of normochromic erythrocytes?
a) High MCV
b) Low MCHC
c) Normal MCHC
d) Elevated MCV
View Answer

c) Normal MCHC

 

20. Which type of anemia is associated with pernicious anemia?
a) Microcytic hypochromic anemia
b) Normocytic normochromic anemia
c) Macrocytic normochromic anemia
d) Hyperchromic anemia
View Answer

c) Macrocytic normochromic anemia

 

21. What term describes erythrocytes with a low mean corpuscular volume (MCV) and normal mean corpuscular hemoglobin concentration (MCHC)?
a) Microcytic normochromic
b) Microcytic hypochromic
c) Macrocytic normochromic
d) Normocytic normochromic
View Answer

b) Microcytic hypochromic

 

22. Which type of anemia is characterized by both low MCV and MCHC?
a) Iron deficiency anemia
b) Pernicious anemia
c) Renal anemia
d) Thalassemia
View Answer

d) Thalassemia

 

23. What is the primary cause of iron deficiency anemia?
a) Excessive iron intake
b) Vitamin B12 deficiency
c) Decreased iron absorption
d) Autoimmune disorder
View Answer

c) Decreased iron absorption

 

24. Which type of anemia is caused by a deficiency in intrinsic factor?
a) Iron deficiency anemia
b) Renal anemia
c) Pernicious anemia
d) Thalassemia
View Answer

c) Pernicious anemia

 

25. What nutrient is essential for normal red blood cell production and maturation in pernicious anemia?
a) Iron
b) Vitamin B12
c) Folate
d) Vitamin C
View Answer

b) Vitamin B12

 

26. Which factor is necessary for the absorption of vitamin B12 from the intestinal tract?
a) Intrinsic factor
b) Hemoglobin
c) Erythropoietin
d) Gastrin
View Answer

a) Intrinsic factor

 

27. What is the primary stimulus for promoting erythropoiesis?
a) Erythropoietin
b) Iron
c) Folic acid
d) Vitamin B12
View Answer

a) Erythropoietin

 

28. Which type of anemia may result from kidney disease?
a) Iron deficiency anemia
b) Pernicious anemia
c) Renal anemia
d) Thalassemia
View Answer

c) Renal anemia

 

29. In renal anemia, inadequate erythropoietin secretion leads to:
a) Increased RBC production
b) Decreased RBC production
c) Normal RBC production
d) Abnormal RBC morphology
View Answer

b) Decreased RBC production

 

30. Which anemia type is associated with both low MCV and low MCHC?
a) Microcytic normochromic
b) Normocytic normochromic
c) Microcytic hypochromic
d) Hyperchromic macrocytic
View Answer

c) Microcytic hypochromic

 

31. What is the main dietary source of iron?
a) Dairy products
b) Meat and poultry
c) Grains and cereals
d) Fruits and vegetables
View Answer

b) Meat and poultry

 

32. Which type of anemia is characterized by large, immature red blood cells?
a) Iron deficiency anemia
b) Pernicious anemia
c) Renal anemia
d) Aplastic anemia
View Answer

b) Pernicious anemia

 

33. Which condition leads to a deficiency in intrinsic factor?
a) Autoimmunity
b) Gastric bypass surgery
c) Kidney disease
d) Liver dysfunction
View Answer

b) Gastric bypass surgery

 

34. In which anemia type is there a decrease in iron absorption from the diet?
a) Iron deficiency anemia
b) Pernicious anemia
c) Renal anemia
d) Thalassemia
View Answer

a) Iron deficiency anemia

 

35. What is the role of intrinsic factor in vitamin B12 absorption?
a) It transports vitamin B12 across the intestinal lining
b) It enhances erythropoietin secretion
c) It promotes iron absorption
d) It regulates red blood cell production
View Answer

a) It transports vitamin B12 across the intestinal lining

 

36. Which anemia type results from autoimmune destruction of gastric parietal cells?
a) Iron deficiency anemia
b) Pernicious anemia
c) Renal anemia
d) Thalassemia
View Answer

b) Pernicious anemia

 

37. What is the primary cause of vitamin B12 deficiency in pernicious anemia?
a) Excessive dietary intake of vitamin B12
b) Intrinsic factor deficiency
c) Impaired erythropoietin secretion
d) Kidney dysfunction
View Answer

b) Intrinsic factor deficiency

 

38. Which anemia type is associated with ineffective erythropoiesis?
a) Iron deficiency anemia
b) Pernicious anemia
c) Renal anemia
d) Thalassemia
View Answer

d) Thalassemia

 

39. What is the characteristic feature of macrocytic normochromic anemia?
a) Low MCV and low MCHC
b) High MCV and normal MCHC
c) High MCV and high MCHC
d) Normal MCV and high MCHC
View Answer

b) High MCV and normal MCHC

 

40. In which anemia type is there insufficient erythropoietin secretion due to kidney disease?
a) Iron deficiency anemia
b) Pernicious anemia
c) Renal anemia
d) Thalassemia
View Answer

c) Renal anemia

 

41. What is the primary cause of aplastic anemia?
a) Destruction of red bone marrow by toxic chemicals
b) Excessive production of erythrocytes
c) Overconsumption of iron-rich foods
d) Genetic mutation in hemoglobin synthesis
View Answer

a) Destruction of red bone marrow by toxic chemicals

 

42. Which of the following conditions can lead to acute hemorrhagic anemia?
a) Prolonged exposure to radiation
b) Chronic bleeding from peptic ulcers
c) Sudden loss of blood due to trauma
d) Long-term chemotherapy treatment
View Answer

c) Sudden loss of blood due to trauma

 

43. Chronic hemorrhagic anemia is characterized by:
a) A rapid decrease in hemoglobin levels
b) Continuous loss of blood over an extended period
c) Destruction of red blood cells in the spleen
d) Excessive production of platelets in the bone marrow
View Answer

b) Continuous loss of blood over an extended period

 

44. Hemolytic anemia results from:
a) Decreased destruction of red blood cells
b) Abnormal breakdown of red blood cells
c) Increased production of erythropoietin
d) Overactive bone marrow
View Answer

b) Abnormal breakdown of red blood cells

 

45. Which type of hemolytic anemia is caused by malaria and snake venom?
a) Normal cell hemolytic
b) Intravascular hemolysis
c) Extravascular hemolysis
d) Defective cell hemolytic
View Answer

a) Normal cell hemolytic

 

46. Sickle cell anemia and thalassemia are examples of:
a) Intravascular hemolysis
b) Hemorrhagic anemia
c) Defective cell hemolytic anemia
d) Aplastic anemia
View Answer

c) Defective cell hemolytic anemia

 

47. In aplastic anemia, the severity of the condition depends on:
a) The individual’s age
b) The extent of destruction to erythropoietic tissue
c) The presence of genetic mutations
d) The level of circulating hemoglobin
View Answer

b) The extent of destruction to erythropoietic tissue

 

48. Acute hemorrhagic anemia is commonly associated with:
a) Chronic illness
b) Surgical procedures
c) Genetic predisposition
d) Bone marrow failure
View Answer

b) Surgical procedures

 

49. Which factor distinguishes chronic hemorrhagic anemia from acute hemorrhagic anemia?
a) Speed of onset
b) Severity of symptoms
c) Presence of trauma
d) Duration of bleeding
View Answer

d) Duration of bleeding

 

50. The destruction of red blood cells in hemolytic anemia can occur:
a) Only in the spleen
b) Both in the blood vessels and spleen
c) Exclusively in the bone marrow
d) Outside the body
View Answer

b) Both in the blood vessels and spleen

Download Link

Download PDF

Edema

1. What is the main characteristic of edema?
a) Accumulation of proteins in body cavities
b) Accumulation of free fluid in tissue spaces
c) Excessive blood clotting
d) Reduced fluid retention in blood vessels
View Answer

b) Accumulation of free fluid in tissue spaces

 

2. Which type of edema may cause ulcers on the skin due to interference with blood flow?
a) Lymphatic edema
b) Inflammatory edema
c) Cardiac edema
d) Nutritional edema
View Answer

c) Cardiac edema

 

3. In which scenario may tongue or throat edema become life-threatening?
a) Mosquito bite
b) Allergic reaction to medication
c) Renal edema
d) Nutritional edema
View Answer

b) Allergic reaction to medication

 

4. What symptoms may be associated with pulmonary edema?
a) Heavy legs
b) Ulcers on the skin
c) Shortness of breath
d) Low albumin levels
View Answer

c) Shortness of breath

 

5. What condition is characterized by the weakening of the heart’s pumping ability?
a) Liver disease
b) Renal failure
c) Congestive heart failure
d) Allergic reaction
View Answer

c) Congestive heart failure

 

6. Which of the following is NOT a cause of edema?
a) Low albumin
b) Allergic reactions
c) High blood pressure
d) Obstruction of flow
View Answer

d) Obstruction of flow

 

7. What can cause edema all over the body in response to critical illness?
a) Excessive albumin levels
b) Blood clot in the arteries
c) Leakage of fluid into tissues
d) Improved fluid drainage
View Answer

c) Leakage of fluid into tissues

 

8. Which disease can lead to fluid retention in the abdomen and leg edema?
a) Kidney disease
b) Liver disease
c) Lung disease
d) Heart disease
View Answer

b) Liver disease

 

9. What role do proteins like albumin play in preventing edema?
a) They promote fluid leakage into tissues
b) They act as sponges to keep fluid in blood vessels
c) They stimulate allergic reactions
d) They obstruct fluid flow
View Answer

b) They act as sponges to keep fluid in blood vessels

 

10. Which type of edema is associated with burns and life-threatening infections?
a) Lymphatic edema
b) Inflammatory edema
c) Renal edema
d) Nutritional edema
View Answer

b) Inflammatory edema

 

11. Which kidney condition can lead to severe leg edema?
a) Glomerulonephritis
b) Nephrotic syndrome
c) Renal artery stenosis
d) Polycystic kidney disease
View Answer

b) Nephrotic syndrome

 

12. How can a doctor diagnose edema?
a) Blood test
b) X-ray
c) Physical examination
d) MRI scan
View Answer

c) Physical examination

 

13. What characteristic of the skin may indicate edema?
a) Dry and flaky
b) Red and inflamed
c) Stretched and shiny
d) Cool to the touch
View Answer

c) Stretched and shiny

 

14. Which action can leave a dimple on the swollen area if someone has edema?
a) Pressing gently for about 15 seconds
b) Rubbing vigorously
c) Applying heat
d) Elevating the limb
View Answer

a) Pressing gently for about 15 seconds

 

15. Which type of medications can cause edema?
a) Antibiotics
b) NSAIDs
c) Antidepressants
d) Antihistamines
View Answer

b) NSAIDs

 

16. What is the effect of pushing gently on the swollen area in edema?
a) It increases swelling
b) It decreases swelling
c) It leaves a dimple
d) It changes the color of the skin
View Answer

c) It leaves a dimple

 

17. Which of the following is NOT a symptom of edema?
a) Shortness of breath
b) Swollen ankles
c) Dimpled skin
d) Elevated blood pressure
View Answer

d) Elevated blood pressure

 

18. What is the term for leg swelling associated with kidney disease?
a) Lymphedema
b) Nephrosis
c) Nephrotic edema
d) Pedal edema
View Answer

d) Pedal edema

 

19. Which class of drugs includes ibuprofen and naproxen?
a) Antibiotics
b) Antidepressants
c) NSAIDs
d) Steroids
View Answer

c) NSAIDs

 

20. What type of kidney condition is characterized by proteinuria and edema?
a) Nephritis
b) Nephrotic syndrome
c) Pyelonephritis
d) Glomerulonephritis
View Answer

b) Nephrotic syndrome

Download Link

Download PDF

Cardiovascular Diseases

1. What is the primary cause of traumatic pericarditis in cattle?
a) Ingestion of sharp foreign objects
b) Bacterial infections
c) Autoimmune disorders
d) Heart attack
View Answer

a) Ingestion of sharp foreign objects

2. How do cattle commonly ingest foreign objects leading to traumatic pericarditis?
a) Through direct swallowing of metal materials
b) Discriminating metal materials in their feed
c) Complete mastication of feed before swallowing
d) Falling directly into the rumen
View Answer

a) Through direct swallowing of metal materials

3. Which factor promotes the penetration of the reticular wall by foreign objects in cattle?
a) Lack of pressure during late pregnancy
b) Absence of efforts during parturition
c) Contractions of the reticulum
d) Passive digestion process
View Answer

c) Contractions of the reticulum

4. What is a common symptom of pericarditis?
a) Dizziness
b) Blurred vision
c) Sudden onset of sharp chest pain
d) Nausea
View Answer

c) Sudden onset of sharp chest pain

5. How does the severity of chest pain in pericarditis typically vary?
a) Less severe when lying down
b) More severe when sitting up
c) Unaffected by body position
d) More severe during exercise
View Answer

b) More severe when sitting up

6. Which condition may produce similar symptoms to pericarditis?
a) Myocarditis
b) Pneumonia
c) Asthma
d) Gastritis
View Answer

a) Myocarditis

7. What is the primary treatment for pericarditis?
a) Antibiotics
b) NSAIDs and colchicine
c) Chemotherapy
d) Surgery
View Answer

b) NSAIDs and colchicine

8. What is a potential complication of pericarditis?
a) Hypertension
b) Renal failure
c) Cardiac tamponade
d) Gastrointestinal bleeding
View Answer

c) Cardiac tamponade

9. Which term refers to inflammation of the heart muscle?
a) Pericarditis
b) Endocarditis
c) Myocarditis
d) Cardiomyopathy
View Answer

c) Myocarditis

10. What is a common symptom of myocarditis?
a) Fever
b) Headache
c) Decreased appetite
d) Shortness of breath
View Answer

d) Shortness of breath

11. What is the most common cause of myocarditis?
a) Bacterial infections
b) Heart attack
c) Autoimmune disorders
d) Viral infection
View Answer

d) Viral infection

12. Which condition may lead to heart failure or cardiac arrest as complications?
a) Pericarditis
b) Tuberculosis
c) Myocarditis
d) Uremic pericarditis
View Answer

c) Myocarditis

13. What type of medication may cause myocarditis?
a) NSAIDs
b) Antibiotics
c) Chemotherapy drugs
d) Antihistamines
View Answer

c) Chemotherapy drugs

14. What is a potential cause of traumatic pericarditis besides sharp foreign objects?
a) Allergic reactions
b) Chemical exposure
c) Muscular strain
d) Heart valve abnormalities
View Answer

b) Chemical exposure

15. What is the role of the reticulum in the development of traumatic pericarditis?
a) Acts as a barrier to foreign objects
b) Facilitates digestion of sharp materials
c) Contracts to promote penetration by foreign objects
d) Provides cushioning for the heart
View Answer

c) Contracts to promote penetration by foreign objects

16. What diagnostic tool is essential for confirming pericarditis?
a) MRI
b) X-ray
c) Electrocardiogram
d) Blood test
View Answer

c) Electrocardiogram

17. What distinguishes the onset of symptoms in pericarditis from a heart attack?
a) Gradual onset
b) Fever and chills
c) Location of pain
d) Presence of chest tightness
View Answer

a) Gradual onset

18. What is the primary aim of using steroids in the treatment of pericarditis?
a) Reduce inflammation
b) Prevent infection
c) Improve blood circulation
d) Alleviate pain
View Answer

a) Reduce inflammation

19. What is the potential consequence of untreated pericarditis?
a) Respiratory failure
b) Liver damage
c) Constrictive pericarditis
d) Gastrointestinal bleeding
View Answer

c) Constrictive pericarditis

20. What distinguishes the pain of pericarditis from that of myocarditis?
a) Location and intensity
b) Duration and frequency
c) Response to medication
d) Associated symptoms
View Answer

a) Location and intensity

21. What imaging technique is essential for ruling out other potential causes such as heart valve problems?
a) Electrocardiogram
b) Heart MRI
c) Ultrasound of the heart
d) Heart biopsy
View Answer

c) Ultrasound of the heart

22. In severe cases of endocarditis, which intervention might be recommended?
a) Heart MRI
b) Heart biopsy
c) Implantable cardiac defibrillator
d) Increased troponin levels
View Answer

c) Implantable cardiac defibrillator

23. Which of the following is NOT a sign or symptom of endocarditis?
a) Splenomegaly
b) Malaise
c) Hypertension
d) Janeway lesions
View Answer

c) Hypertension

24. What is the characteristic lesion in endocarditis composed of?
a) Red blood cells
b) White blood cells
c) Platelets, fibrin, microorganisms, and inflammatory cells
d) Granulomatous tissue
View Answer

c) Platelets, fibrin, microorganisms, and inflammatory cells

25. Which of the following is NOT mentioned as a treatment option for endocarditis?
a) Corticosteroids
b) ACE inhibitors
c) Heart transplant
d) Antibiotics
View Answer

b) ACE inhibitors

26. What diagnostic tool is primarily used to demonstrate the presence of endocarditis-causing microorganisms?
a) Blood cultures
b) Heart MRI
c) Heart biopsy
d) Ultrasound of the heart
View Answer

a) Blood cultures

27. Which structure of the heart is primarily affected by endocarditis?
a) Myocardium
b) Epicardium
c) Endocardium
d) Pericardium
View Answer

c) Endocardium

28. What might be included in the vegetation of subacute infective endocarditis?
a) Platelets only
b) Microorganisms only
c) Granulomatous tissue
d) Fibrin and microcolonies only
View Answer

c) Granulomatous tissue

29. Which of the following is NOT a potential cause of endocarditis?
a) Infective microorganisms
b) Non-infective sources
c) Heart valve problems
d) Hypertension
View Answer

d) Hypertension

30. What is the recommended period of rest during recovery from endocarditis?
a) No rest required
b) Light exercise allowed
c) A period of no exercise
d) Intensive exercise regimen
View Answer

c) A period of no exercise

31. Which of the following is NOT a typical sign of endocarditis?
a) Heart failure
b) Splenomegaly
c) Hyperactivity
d) Petechiae
View Answer

c) Hyperactivity

32. What might be found on the palms and soles of a patient with endocarditis?
a) Janeway lesions
b) Osler’s nodes
c) Roth’s spots
d) Petechiae
View Answer

a) Janeway lesions

33. What is the characteristic composition of Osler’s nodes?
a) Microorganisms
b) Fibrin
c) Subcutaneous nodules
d) Retinal hemorrhages
View Answer

c) Subcutaneous nodules

34. What is the primary method for diagnosing endocarditis?
a) Electrocardiogram
b) Blood cultures
c) Heart MRI
d) Troponin levels
View Answer

b) Blood cultures

35. What is the primary classification of endocarditis based on?
a) Severity
b) Presence of microorganisms
c) Type of lesions
d) Location in the heart
View Answer

View AnswerText
[/expand]

36. What is the characteristic feature of Roth’s spots?
a) Subcutaneous nodules
b) Petechiae
c) Retinal hemorrhages
d) Palmar lesions
View Answer

c) Retinal hemorrhages

37. Which medication is NOT commonly used in the treatment of endocarditis?
a) ACE inhibitors
b) Diuretics
c) Corticosteroids
d) Antibiotics
View Answer

a) ACE inhibitors

38. What may be present in the vegetation of endocarditis lesions?
a) Microorganisms only
b) Platelets and fibrin only
c) Granulomatous tissue
d) Red blood cells
View Answer

c) Granulomatous tissue

39. What is the typical involvement of endocarditis in the heart?
a) Myocardium
b) Epicardium
c) Endocardium
d) Pericardium
View Answer

c) Endocardium

40. Which of the following is NOT a potential symptom of endocarditis?
a) Malaise
b) Fever
c) Hypertension
d) Weight loss
View Answer

c) Hypertension

41. Which term refers to the inability of the heart to pump sufficient blood to meet the needs of the tissues for oxygen and nutrients?
a) Acute heart failure
b) Congestive heart failure
c) Myocardial disease
d) Decompensatio cordis
View Answer

a) Acute heart failure

42. Which of the following cardiovascular conditions can contribute to the development of heart failure?
a) Chronic hypertension
b) Hypothyroidism
c) Diabetes mellitus
d) Osteoarthritis
View Answer

a) Chronic hypertension

43. What is the primary pathophysiological mechanism underlying heart failure?
a) Increased cardiac output
b) Decreased myocardial dysfunction
c) Impaired cardiac pumping
d) Elevated tissue perfusion
View Answer

c) Impaired cardiac pumping

44. Which symptom is typically worse with exercise or while lying down in patients with heart failure?
a) Chest pain
b) Leg swelling
c) Excessive tiredness
d) Shortness of breath
View Answer

d) Shortness of breath

45. What is a common feature of heart failure related to the patient’s ability to exercise?
a) Enhanced exercise tolerance
b) Unaffected exercise capacity
c) Limited ability to exercise
d) Improved exercise performance
View Answer

c) Limited ability to exercise

46. What is the primary goal of medical management in heart failure?
a) Reduce patient symptoms
b) Improve functional status
c) Extend survival
d) All of the above
View Answer

d) All of the above

47. Which treatment option may involve the manipulation of assistive devices in patients with heart failure?
a) Lifestyle changes
b) Pharmacologic management
c) Supplemental oxygen
d) Surgical approaches
View Answer

d) Surgical approaches

48. What specific objective of treatment aims to eliminate or reduce reversible contributing factors to heart failure?
a) Relieve patient symptoms
b) Reduce workload of the heart
c) Optimize therapeutic regimen
d) Prevent exacerbations
View Answer

b) Reduce workload of the heart

49. Which surgical approach involves rerouting blood flow around blocked coronary arteries to improve blood supply to the heart muscle?
a) Open heart surgery
b) Heart transplantation
c) Coronary artery bypass grafting (CABG)
d) Ventricular assist device (VAD) implantation
View Answer

c) Coronary artery bypass grafting (CABG)

50. What is the primary cause of pulmonary or systemic congestion in heart failure?
a) Elevated cardiac output
b) Increased tissue perfusion
c) Decreased contraction of the heart
d) Impaired diastolic filling
View Answer

c) Decreased contraction of the heart

51. Which of the following is NOT a sign or symptom commonly associated with heart failure?
a) Excessive tiredness
b) Leg swelling
c) Chest pain
d) Shortness of breath
View Answer

c) Chest pain

52. In heart failure, what condition results from the heart’s inability to generate sufficient cardiac output to meet the body’s demands?
a) Hypoxemia
b) Fluid overload
c) Tissue hypoperfusion
d) Hyperglycemia
View Answer

c) Tissue hypoperfusion

53. What cardiovascular condition contributes to decreased filling (diastole) of the heart in heart failure?
a) Chronic hypertension
b) Peripheral artery disease
c) Aortic stenosis
d) Pulmonary embolism
View Answer

c) Aortic stenosis

54. Which of the following is a primary characteristic of congestive heart failure?
a) Reduced myocardial dysfunction
b) Increased tissue perfusion
c) Pulmonary or systemic congestion
d) Enhanced cardiac pumping
View Answer

c) Pulmonary or systemic congestion

55. Which medical management approach aims to optimize the therapeutic regimen for patients with heart failure?
a) Lifestyle changes
b) Pharmacologic management
c) Preventing exacerbations
d) Reducing workload of the heart
View Answer

b) Pharmacologic management

56. What type of surgery involves the replacement of a damaged or failing heart with a healthy donor heart?
a) Open heart surgery
b) Coronary artery bypass grafting (CABG)
c) Heart transplantation
d) Ventricular assist device (VAD) implantation
View Answer

c) Heart transplantation

57. Which symptom is commonly worse at night and may wake the person affected by heart failure?
a) Leg swelling
b) Excessive tiredness
c) Shortness of breath
d) Chest pain
View Answer

c) Shortness of breath

58. What is the primary factor contributing to the limited ability to exercise in patients with heart failure?
a) Increased cardiac output
b) Enhanced tissue perfusion
c) Decreased myocardial dysfunction
d) Impaired cardiac pumping
View Answer

d) Impaired cardiac pumping

59. What term refers to the condition when the heart is unable to pump sufficiently to maintain blood flow to meet the body tissues’ needs for metabolism?
a) Decompensatio cordis
b) Chronic hypertension
c) Congestive heart failure
d) Hyperglycemia
View Answer

a) Decompensatio cordis

60. Which lifestyle modification is commonly recommended for patients with heart failure?
a) Smoking cessation
b) Increased alcohol consumption
c) Sedentary lifestyle
d) High-sodium diet
View Answer

a) Smoking cessation

Download Link

Download PDF

Animal Restraining Techniques

1. What is the primary purpose of animal restraint in veterinary procedures?
A) To induce fear in the animal
B) To ensure the safety of the animal and handlers
C) To maximize aggression in the animal
D) To minimize the effectiveness of the procedure
View Answer

B) To ensure the safety of the animal and handlers

 

2. Which of the following is NOT a type of restraint technique?
A) Psychological
B) Physical
C) Chemical
D) Environmental
View Answer

A) Psychological

 

3. According to the principles of restraint, what is the essential consideration when using physical restraint?
A) Maximizing the duration of restraint
B) Ensuring the comfort of the animal
C) Prioritizing the safety and dignity of the patient and staff
D) Minimizing staff involvement
View Answer

C) Prioritizing the safety and dignity of the patient and staff

 

4. What are three indicators mentioned in the text for recognizing animal restraint?
A) Avoiding eye contact, hunching posture, whispering
B) Staring directly into its eyes, placing hands on hips, excessive arm movements
C) Speaking softly, avoiding sudden movements, maintaining distance
D) Clapping, jumping, shouting
View Answer

B) Staring directly into its eyes, placing hands on hips, excessive arm movements

 

5. How is livestock often restrained for procedures?
A) Using psychological techniques
B) Utilizing ropes and cords
C) Administering sedatives
D) Restraining with verbal commands
View Answer

B) Utilizing ropes and cords

 

6. Which method is NOT mentioned as a variation for cattle restraint?
A) Nose tongs
B) Hobbling
C) Psychological restraint
D) Flanking
View Answer

C) Psychological restraint

 

7. In dog restraint, what technique involves placing one arm under the dog’s neck while holding the head?
A) Sitting restraint
B) Lateral recumbency
C) Standing restraint
D) Sternal recumbency
View Answer

C) Standing restraint

 

8. What does the “double loop” method mentioned in the text involve?
A) Applying two chemical restraints simultaneously
B) Utilizing two physical restraints in tandem
C) Looping a rope around an animal’s upper jaw
D) Restraining the animal in a confined space
View Answer

C) Looping a rope around an animal’s upper jaw

 

9. What should be the minimum duration for using physical restraint on animals, according to the principles mentioned?
A) As long as necessary
B) Until the animal becomes calm
C) Only for a brief moment
D) Until the animal shows signs of aggression
View Answer

C) Only for a brief moment

 

10. Which technique involves diverting attention in cattle restraint?
A) Nose tongs
B) Manual nose twitch
C) Psychological manipulation
D) Flanking
View Answer

B) Manual nose twitch

 

11. Which position in dog restraint involves the dog lying on its side?
A) Lateral recumbency
B) Standing restraint
C) Sternal recumbency
D) Dorsal recumbency
View Answer

A) Lateral recumbency

 

12. What is the primary consideration when handling laboratory animals?
A) Maximizing stress
B) Minimizing stress
C) Ignoring stress
D) Inducing stress
View Answer

B) Minimizing stress

 

13. What should be considered for prolonged or potentially painful procedures?
A) Increased restraint duration
B) Decreased restraint duration
C) Chemical restraint
D) Ignoring restraint
View Answer

C) Chemical restraint

 

14. Which technique involves threading a rope through an animal’s upper jaw and tying it to a post?
A) Hobbling
B) Flanking
C) Casting
D) Looping
View Answer

D) Looping

 

15. Which technique involves placing a rope around the hind legs of cattle?
A) Flanking
B) Nose tongs
C) Psychological restraint
D) Standing restraint
View Answer

A) Flanking

 

16. What is NOT a position used in dog restraint?
A) Prone
B) Supine
C) Lateral recumbency
D) Sternal recumbency
View Answer

A) Prone

 

17. Which indicator is NOT mentioned as a sign of animal restraint?
A) Excessive arm movements
B) Whispering
C) Placing hands on hips
D) Staring directly into its eyes
View Answer

B) Whispering

 

18. What is the primary purpose of the “double loop” method?
A) To induce fear in the animal
B) To effectively restrain the animal’s movement
C) To mimic psychological restraint
D) To facilitate communication with the animal
View Answer

B) To effectively restrain the animal’s movement

 

19. Which restraint technique involves diverting the attention of the animal?
A) Standing restraint
B) Flanking
C) Manual nose twitch
D) Dorsal recumbency
View Answer

C) Manual nose twitch

 

20. What is the main objective of proper restraint and handling techniques?
A) To increase stress on the animal
B) To decrease stress on the animal and handlers
C) To induce aggression in the animal
D) To ignore the safety of the handlers
View Answer

B) To decrease stress on the animal and handlers

Download Link

Download PDF

General Examination

1. When conducting a general inspection of an animal, which of the following is NOT considered?
A) Behavior
B) Coat color and markings
C) Temperature of the environment
D) General body condition
View Answer

C) Temperature of the environment

 

2. Which behavior may indicate an abnormal condition in an animal?
A) Increased appetite
B) Restlessness
C) Decreased thirst
D) Calm demeanor
View Answer

B) Restlessness

 

3. What aspect of an animal’s appearance should a veterinarian note to identify it?
A) Breed
B) Eye color
C) Coat color and markings
D) Tail length
View Answer

C) Coat color and markings

 

4. How can the general condition of an animal be assessed?
A) By checking the temperature of the animal’s body
B) By examining specific body parts such as ribs and pelvis
C) By observing the animal’s behavior only
D) By listening to its heartbeat
View Answer

B) By examining specific body parts such as ribs and pelvis

 

5. Abnormal posture in animals may indicate problems with which body parts?
A) Skin and fur
B) Bones, joints, ligaments, and tendons
C) Digestive system
D) Respiratory system
View Answer

B) Bones, joints, ligaments, and tendons

 

6. What does the term “gait” refer to in veterinary medicine?
A) Animal’s behavior while interacting with other animals
B) Animal’s method of communication
C) Animal’s movement, including the locomotor system
D) Animal’s feeding behavior
View Answer

C) Animal’s movement, including the locomotor system

 

7. How is palpation performed in veterinary examination?
A) By using X-rays
B) By observing the animal’s behavior
C) By touching and feeling the animal’s body
D) By measuring the animal’s temperature
View Answer

C) By touching and feeling the animal’s body

 

8. Which method of palpation involves the use of an instrument?
A) Direct method
B) Indirect method
C) Both direct and indirect methods
D) Neither direct nor indirect methods
View Answer

B) Indirect method

 

9. When is auscultation performed directly?
A) When using a stethoscope
B) When observing the animal’s breathing
C) When listening to sounds without any instrument
D) When feeling the animal’s body
View Answer

C) When listening to sounds without any instrument

 

10. Which tool is used in the direct method of auscultation?
A) Stethoscope
B) Otoscope
C) Thermometer
D) Sphygmomanometer
View Answer

D) Sphygmomanometer[/expand]

 

11. Which condition might be detected through direct auscultation?
A) Bladder stones
B) Intestinal obstruction
C) Respiratory infection
D) Heart murmur
View AnswerC) Respiratory infection

 

12. In which method of palpation is a stomach tube used?
A) Direct method
B) Indirect method
C) Both direct and indirect methods
D) Neither direct nor indirect methods
View Answer

B) Indirect method

 

13. What might be examined using direct palpation?
A) Lymph nodes
B) Internal organs
C) Coat color
D) Behavior
View Answer

A) Lymph nodes

 

14. How is indirect palpation performed?
A) By using a stethoscope
B) By using fingers to feel the body
C) By using instruments to feel internal organs
D) By observing the animal’s behavior
View Answer

C) By using instruments to feel internal organs

 

15. Which method is used to examine superficial lymph nodes?
A) Direct palpation
B) Indirect palpation
C) Percussion
D) Auscultation
View Answer

A) Direct palpation

 

16. Which of the following is NOT a part of indirect palpation?
A) Using a stomach tube
B) Feeling the abdomen
C) Listening to sounds
D) Using fingers to feel the body
View Answer

D) Using fingers to feel the body

 

17. Which of the following is NOT a sign of abnormal behavior in an animal?
A) Increased thirst
B) Restlessness
C) Kicking the abdomen
D) Difficulty in walking
View Answer

A) Increased thirst

 

18. Which of the following is NOT examined during general inspection of an animal?
A) Behavior
B) Coat color
C) Internal organs
D) General body condition
View Answer

C) Internal organs

 

19. Which posture might indicate problems with bone, joint, ligament, or tendon?
A) Standing upright
B) Laying down
C) Hunched over
D) Walking slowly
View Answer

C) Hunched over

 

20. What might be examined through indirect auscultation?
A) Heart sounds
B) Respiratory sounds
C) Intestinal sounds
D) All of the above
View Answer

D) All of the above

Download Link

Download PDF

 

History Taking Of Animals

1. What is the primary reason why history-taking is crucial in veterinary medicine?
a) To understand the animal’s clinical symptoms
b) To assess the accuracy of the owner’s observations
c) To determine the breed of the animal
d) To dispatch specimens for laboratory examination
View Answer

b) To assess the accuracy of the owner’s observations

 

2. Why is the use of nontechnical terms essential in communication with livestock owners?
a) To confuse owners
b) To express technical expressions
c) To ensure clarity and understanding
d) To discourage owners from expressing themselves
View Answer

c) To ensure clarity and understanding

 

3. What should a veterinarian do to verify the accuracy of the history provided by the owner?
a) Trust the owner’s words without examination
b) Conduct laboratory tests immediately
c) Examine the animal carefully
d) Consult with other veterinarians
View Answer

c) Examine the animal carefully

 

4. Which of the following is NOT a relevant piece of data in patient history?
a) Owner’s name and initials
b) Species type or breed
c) Owner’s favorite color
d) Bodyweight
View Answer

c) Owner’s favorite color

 

5. What does the morbidity rate indicate?
a) Percentage of animals that die
b) Percentage of animals that are clinically affected
c) Percentage of animals with previous exposure
d) Percentage of animals with previous disease
View Answer

b) Percentage of animals that are clinically affected

 

6. Why is the history of the group relative to additions important in disease examination?
a) To know the breeding policy
b) To understand housing conditions
c) To assess previous exposure to disease
d) To determine the owner’s name
View Answer

c) To assess previous exposure to disease

 

7. What is the significance of the management history in veterinary medicine?
a) To understand the owner’s profession
b) To determine the animal’s favorite food
c) To learn about nutrition, breeding, housing, and handling
d) To examine the owner’s transportation habits
View Answer

c) To learn about nutrition, breeding, housing, and handling

 

8. What is the primary objective of examining the nutritional history of animals?
a) To determine the breed
b) To assess the owner’s diet
c) To compare the animal’s diet with recommended nutrient requirements
d) To calculate the population mortality rate
View Answer

c) To compare the animal’s diet with recommended nutrient requirements

 

9. In disease history, what does the case fatality rate indicate?
a) Percentage of animals that die
b) Percentage of affected animals that die
c) Percentage of animals with previous exposure
d) Percentage of animals with previous disease
View Answer

b) Percentage of affected animals that die

 

10. Why is it essential to examine the remainder of a group when one animal is affected by a disease?
a) To determine the breed of other animals
b) To identify early stages of the disease in other animals
c) To assess the population mortality rate
d) To interview other owners
View Answer

b) To identify early stages of the disease in other animals

 

11. What should a veterinarian do if there is a history of previous illness in a group of animals?
a) Ignore the history
b) Investigate the previous illness and treatments used
c) Immediately vaccinate all animals
d) Recommend euthanasia for all animals
View Answer

b) Investigate the previous illness and treatments used

 

12. What is the primary reason for using nontechnical terms when communicating with livestock owners?
a) To confuse owners
b) To avoid expressing technical expressions
c) To ensure clarity and understanding
d) To discourage owners from expressing themselves
View Answer

c) To ensure clarity and understanding

 

13. What information is crucial to determine from the management history prior to the appearance of disease?
a) The owner’s favorite color
b) Any changes in prevailing practices
c) The breed of the animals
d) The transportation habits of the owner
View Answer

b) Any changes in prevailing practices

 

14. How can veterinarians assess the accuracy of the history provided by the owner?
a) Trust the owner’s words without examination
b) Conduct laboratory tests immediately
c) Examine the animal carefully
d) Consult with other veterinarians
View Answer

c) Examine the animal carefully

 

15. What is the primary objective of examining the nutritional history of animals?
a) To determine the breed
b) To assess the owner’s diet
c) To compare the animal’s diet with recommended nutrient requirements
d) To calculate the population mortality rate
View Answer

c) To compare the animal’s diet with recommended nutrient requirements

 

16. Why is it important to examine the remainder of a group when one animal is affected by a disease?
a) To determine the breed of other animals
b) To identify early stages of the disease in other animals
c) To assess the population mortality rate
d) To interview other owners
View Answer

b) To identify early stages of the disease in other animals

 

17. What information is crucial to determine from the management history prior to the appearance of disease?
a) The owner’s favorite color
b) Any changes in prevailing practices
c) The breed of the animals
d) The transportation habits of the owner
View Answer

b) Any changes in prevailing practices

 

18. How can veterinarians assess the accuracy of the history provided by the owner?
a) Trust the owner’s words without examination
b) Conduct laboratory tests immediately
c) Examine the animal carefully
d) Consult with other veterinarians
View Answer

c) Examine the animal carefully

 

19. What is the primary objective of examining the nutritional history of animals?
a) To determine the breed
b) To assess the owner’s diet
c) To compare the animal’s diet with recommended nutrient requirements
d) To calculate the population mortality rate
View Answer

c) To compare the animal’s diet with recommended nutrient requirements

 

20. Why is it important to examine the remainder of a group when one animal is affected by a disease?
a) To determine the breed of other animals
b) To identify early stages of the disease in other animals
c) To assess the population mortality rate
d) To interview other owners
View Answer

b) To identify early stages of the disease in other animals

Download Link

Download PDF

 

Methods Of Drug Andministration

1. Which route of drug administration bypasses the gastrointestinal tract entirely?
A) Oral
B) Subcutaneous
C) Intravenous
D) Rectal
View Answer

C) Intravenous

 

2. What is the primary advantage of sublingual drug administration?
A) Rapid absorption
B) Avoidance of first-pass metabolism
C) Minimization of infection risk
D) Controlled release of drug
View Answer

A) Rapid absorption

 

3. Which route of administration is recommended for drugs that require rapid onset and continuous administration?
A) Intramuscular
B) Oral
C) Subcutaneous
D) Intravenous
View Answer

D) Intravenous

 

4. What is a disadvantage of intramuscular injections compared to subcutaneous injections?
A) Slower absorption
B) Higher risk of contamination
C) Greater risk of adverse effects
D) Inability to administer large volumes
View Answer

A) Slower absorption

 

5. Which route of administration is most suitable for asthmatic drugs and anesthetics?
A) Oral
B) Inhalation
C) Intranasal
D) Subcutaneous
View Answer

B) Inhalation

 

6. What characteristic makes rectal administration unreliable?
A) Rapid absorption
B) Bypass of portal circulation
C) Variable drainage patterns
D) High bioavailability
View Answer

C) Variable drainage patterns

 

7. Which route of administration involves placing the drug under the tongue?
A) Oral
B) Subcutaneous
C) Intravenous
D) Sublingual
View Answer

D) Sublingual

 

8. Which route of administration is suitable for patients unable to take medications orally?
A) Intramuscular
B) Subcutaneous
C) Rectal
D) Intranasal
View Answer

C) Rectal

 

9. What makes intravenous administration the fastest and most certain route?
A) Avoidance of first-pass metabolism
B) Bypass of absorption barriers
C) Controlled release of drug
D) Low risk of contamination
View Answer

B) Bypass of absorption barriers

 

10. Which route of administration is used for drugs poorly absorbed or unstable in the gastrointestinal tract?
A) Intravenous
B) Oral
C) Subcutaneous
D) Inhalation
View Answer

A) Intravenous

 

11. Which route of administration involves drug delivery through the skin?
A) Topical
B) Transdermal
C) Intramuscular
D) Intranasal
View Answer

B) Transdermal

 

12. Which route of administration involves injection into a muscle?
A) Intravenous
B) Oral
C) Intramuscular
D) Sublingual
View Answer

C) Intramuscular

 

13. What characteristic distinguishes subcutaneous injections from intramuscular injections?
A) Rapid absorption
B) Slower absorption
C) Risk of contamination
D) Greater volume capacity
View Answer

B) Slower absorption

 

14. Which route of administration is suitable for producing local or systemic effects?
A) Subcutaneous
B) Intravenous
C) Inhalation
D) Rectal
View Answer

D) Rectal

 

15. Which route of administration involves drug delivery into the cerebrospinal fluid?
A) Intravenous
B) Topical
C) Intrathecal
D) Intramuscular
View Answer

C) Intrathecal

 

16. What is the primary advantage of intravenous administration?
A) Controlled release of drug
B) Avoidance of first-pass metabolism
C) Slow absorption
D) Ability to administer large volumes
View Answer

B) Avoidance of first-pass metabolism

 

17. Which route of administration is used for drugs that require sustained release over an extended period?
A) Intravenous
B) Oral
C) Subcutaneous
D) Inhalation
View Answer

C) Subcutaneous

 

18. What distinguishes intrathecal administration from other routes?
A) Drug delivery into the bloodstream
B) Drug delivery into the spinal canal
C) Drug delivery through the skin
D) Drug delivery into the gastrointestinal tract
View Answer

B) Drug delivery into the spinal canal

 

19. Which route of administration is most appropriate for patients experiencing vomiting?
A) Intravenous
B) Rectal
C) Sublingual
D) Intramuscular
View Answer

B) Rectal

 

20. Which route of administration involves drug delivery directly into the nose?
A) Topical
B) Transdermal
C) Intranasal
D) Inhalation
View Answer

C) Intranasal

Download Link

Download PDF

Diseases Of Musculoskeletal System

1. What is the primary function of the musculoskeletal system?
a) Providing oxygen to the body
b) Supporting and protecting organs
c) Regulating body temperature
d) Digesting food
View Answer

b) Supporting and protecting organs

 

2. Which component of the musculoskeletal system stores the majority of calcium in the body?
a) Muscles
b) Ligaments
c) Cartilage
d) Bones
View Answer

d) Bones

 

3. What is the function of cartilage in the musculoskeletal system?
a) Producing red blood cells
b) Facilitating movement in joints
c) Absorbing shock and reducing friction
d) Connecting bone to bone
View Answer

c) Absorbing shock and reducing friction

 

4. Which type of muscle is NOT part of the musculoskeletal system?
a) Skeletal
b) Smooth
c) Cardiac
d) Visceral
View Answer

c) Cardiac

 

5. What are ligaments composed of?
a) Collagen and elastic fibers
b) Cartilage and water
c) Proteoglycans and muscle fibers
d) Calcium and phosphorus
View Answer

a) Collagen and elastic fibers

 

6. What is the primary symptom of myositis?
a) Joint pain
b) Muscle weakness
c) Swelling
d) Fatigue
View Answer

b) Muscle weakness

 

7. Which condition causes inflammation in muscles?
a) Arthritis
b) Osteoporosis
c) Myositis
d) Fibromyalgia
View Answer

c) Myositis

 

8. Which type of myositis affects large muscle groups and causes weakness over weeks or months?
a) Dermatomyositis
b) Polymyositis
c) Infectious myositis
d) Traumatic myositis
View Answer

b) Polymyositis

 

9. What are some common symptoms of inflammatory myositis conditions?
a) Joint stiffness
b) Rash
c) Fatigue
d) All of the above
View Answer

d) All of the above

 

10. Which diagnostic test is NOT typically used for myositis?
a) Blood tests
b) MRI scan
c) Electrocardiogram
d) Muscle biopsy
View Answer

c) Electrocardiogram

 

11. What is the first choice of treatment for myositis?
a) Antibiotics
b) Physical therapy
c) Steroids
d) Antiviral drugs
View Answer

c) Steroids

 

12. What is arthritis characterized by?
a) Inflammation of the muscles
b) Swelling of the joints
c) Weakness of the bones
d) Enlargement of ligaments
View Answer

b) Swelling of the joints

 

13. Which type of arthritis is commonly associated with aging?
a) Osteoarthritis
b) Rheumatoid arthritis
c) Psoriatic arthritis
d) Ankylosing spondylitis
View Answer

a) Osteoarthritis

 

14. What are the main symptoms of arthritis?
a) Muscle weakness and fatigue
b) Joint pain and stiffness
c) Headaches and dizziness
d) Skin rash and fever
View Answer

b) Joint pain and stiffness

 

15. What is a common symptom of arthritis associated with the affected joint?
a) Blue discoloration of the skin
b) Cold sensation
c) Warm red skin
d) Numbness
View Answer

c) Warm red skin

 

16. What is the primary function of ligaments in the musculoskeletal system?
a) Connecting bone to bone
b) Contracting muscles
c) Absorbing shock
d) Providing structural support
View Answer

a) Connecting bone to bone

 

17. Which type of joint allows for the most movement?
a) Synovial joint
b) Cartilaginous joint
c) Fibrous joint
d) Immovable joint
View Answer

a) Synovial joint

 

18. What is the composition of normal cartilage?
a) Collagen, water, and proteoglycans
b) Calcium and phosphorus
c) Elastic fibers and muscle tissue
d) Hemoglobin and platelets
View Answer

a) Collagen, water, and proteoglycans

 

19. Which condition primarily affects the muscles and not the joints?
a) Myositis
b) Arthritis
c) Osteoporosis
d) Fibromyalgia
View Answer

a) Myositis

 

20. Which type of muscle is responsible for voluntary movements?
a) Cardiac muscle
b) Smooth muscle
c) Skeletal muscle
d) Visceral muscle
View Answer

c) Skeletal muscle

 

21. Which of the following diagnostic tools is NOT commonly used in the diagnosis of arthritis?
A) Blood tests
B) X-rays
C) Ultrasound
D) MRI
View Answer

C) Ultrasound

 

22. What is the primary function of cartilage in the joints?
A) To produce synovial fluid
B) To absorb pressure and shock
C) To lubricate the joints
D) To protect the bones from infection
View Answer

B) To absorb pressure and shock

 

23. Which type of arthritis involves the immune system attacking the synovium?
A) Osteoarthritis
B) Rheumatoid arthritis
C) Gouty arthritis
D) Psoriatic arthritis
View Answer

B) Rheumatoid arthritis

 

24. Which medication category helps reduce pain but has no effect on inflammation in arthritis?
A) Painkillers
B) NSAIDs
C) Corticosteroids
D) Biologic response modifiers
View Answer

A) Painkillers

 

25. What is the primary cause of osteomyelitis?
A) Trauma to the bone
B) Autoimmune disorder
C) Bacterial infection
D) Vitamin D deficiency
View Answer

C) Bacterial infection

 

26. Which bacteria is frequently responsible for causing osteomyelitis?
A) Streptococcus
B) Escherichia coli
C) Staphylococcus
D) Salmonella
View Answer

C) Staphylococcus

 

27. What is the most common treatment for osteomyelitis?
A) Intravenous antibiotics
B) Physical therapy
C) Corticosteroid injections
D) NSAIDs
View Answer

A) Intravenous antibiotics

 

28. Which symptom is NOT typically associated with osteomyelitis?
A) Fever
B) Joint pain
C) Drainage of pus
D) Muscle weakness
View Answer

D) Muscle weakness

 

29. What is the primary cause of osteodystrophy?
A) Vitamin D deficiency
B) Renal disease
C) Autoimmune disorder
D) Trauma to the bone
View Answer

B) Renal disease

 

30. Which mineral accumulation in the bloodstream contributes to the development of osteodystrophy?
A) Calcium
B) Magnesium
C) Phosphorus
D) Potassium
View Answer

C) Phosphorus

 

31. How is renal osteodystrophy typically diagnosed?
A) X-ray
B) Blood tests
C) MRI
D) Bone biopsy
View Answer

B) Blood tests

 

32. Which treatment is NOT commonly used for osteodystrophy?
A) Medications
B) Dialysis
C) Surgery
D) Radiation therapy
View Answer

D) Radiation therapy

 

33. What is the primary cause of osteomalacia?
A) Calcium deficiency
B) Phosphorus deficiency
C) Vitamin D deficiency
D) Iron deficiency
View Answer

C) Vitamin D deficiency

 

34. Which population group is most at risk for developing osteomalacia?
A) Elderly adults
B) Teenagers
C) Pregnant women
D) Athletes
View Answer

A) Elderly adults

 

35. What symptom distinguishes osteomalacia from osteomyelitis?
A) Fever
B) Bone pain
C) Drainage of pus
D) Joint deformities
View Answer

B) Bone pain

 

36. Which bone condition involves a marked softening of the bones?
A) Osteoarthritis
B) Osteomyelitis
C) Osteodystrophy
D) Osteomalacia
View Answer

D) Osteomalacia

 

37. What is the primary treatment for osteomalacia?
A) Painkillers
B) Intravenous antibiotics
C) Vitamin D supplementation
D) Biologic response modifiers
View Answer

C) Vitamin D supplementation

 

38. Which symptom is common to both osteodystrophy and osteomalacia?
A) Joint pain
B) Fever
C) Drainage of pus
D) Muscle weakness
View Answer

D) Muscle weakness

 

39. Which diagnostic tool is NOT commonly used in the diagnosis of osteomalacia?
A) X-ray
B) Blood tests
C) CT scan
D) MRI
View Answer

C) CT scan

 

40. Which of the following is NOT a potential cause of osteomyelitis?
A) Recent surgery
B) Bacterial infection
C) Vitamin D deficiency
D) Open wound over a bone
View Answer

C) Vitamin D deficiency

 

41. Which of the following is NOT a symptom of osteomalacia?
a) Joint pain
b) Muscle weakness
c) Bone fractures
d) Increased bone density
View Answer

d) Increased bone density

 

42. Osteomalacia primarily develops due to a deficiency in which vitamin?
a) Vitamin A
b) Vitamin B12
c) Vitamin C
d) Vitamin D
View Answer

d) Vitamin D

 

43. What role does vitamin D play in bone health?
a) Enhances muscle strength
b) Stimulates calcium absorption
c) Promotes joint flexibility
d) Reduces bone density
View Answer

b) Stimulates calcium absorption

 

44. Which of the following tests is NOT used in the diagnosis of osteomalacia?
a) Blood test
b) Urine test
c) MRI scan
d) X-ray
View Answer

c) MRI scan

 

45. What structural changes are characteristic of osteomalacia on X-rays?
a) Bone spurs
b) Visible cracks
c) Cartilage erosion
d) Increased bone density
View Answer

b) Visible cracks

 

46. In a bone biopsy for osteomalacia, where is the sample typically taken from?
a) Arm bone
b) Leg bone
c) Pelvic bone
d) Rib bone
View Answer

c) Pelvic bone

 

47. Which of the following is a common treatment for osteomalacia?
a) Physical therapy
b) Surgery
c) Vitamin D supplements
d) Antibiotics
View Answer

c) Vitamin D supplements

 

48. Apart from vitamin D deficiency, what other disorder can contribute to osteomalacia?
a) Hypertension
b) Diabetes
c) Kidney disorder
d) Thyroid dysfunction
View Answer

c) Kidney disorder

 

49. What is the recommended duration of oral vitamin D supplements to cure osteomalacia?
a) Several days
b) Several weeks to months
c) One year
d) Indefinitely
View Answer

b) Several weeks to months

 

50. How does treating underlying conditions like kidney disease help improve osteomalacia symptoms?
a) Increases bone density
b) Stimulates muscle growth
c) Enhances vitamin D absorption
d) Corrects abnormalities in vitamin D metabolism
View Answer

d) Corrects abnormalities in vitamin D metabolism

Download Link

Download PDF

 

Diseases Of The Nervous System

1. Which division of the nervous system consists of the brain and spinal cord?
a) Peripheral nervous system
b) Central nervous system
c) Autonomic nervous system
d) Somatic nervous system
View Answer

b) Central nervous system

 

2. Which of the following is NOT considered a principal organ of the nervous system?
a) Brain
b) Spinal cord
c) Eyes
d) Liver
View Answer

d) Liver

 

3. Which disorder involves inflammation of the fluid and membranes surrounding the brain and spinal cord?
a) Epilepsy
b) Meningitis
c) Stroke
d) Neuralgia
View Answer

b) Meningitis

 

4. What is the most common cause of meningitis in the United States?
a) Viral infection
b) Bacterial infection
c) Fungal infection
d) Parasitic infection
View Answer

a) Viral infection

 

5. Which bacterium is NOT commonly associated with causing acute bacterial meningitis?
a) Streptococcus pneumoniae
b) Neisseria meningitidis
c) Haemophilus influenzae
d) Escherichia coli
View Answer

d) Escherichia coli

 

6. Which of the following symptoms is NOT typically associated with meningitis?
a) Stiff neck
b) Rash
c) Severe headache
d) Sensitivity to light
View Answer

b) Rash

 

7. What type of infection can lead to acute bacterial meningitis when bacteria directly invade the meninges?
a) Bloodstream infection
b) Respiratory infection
c) Gastrointestinal infection
d) Urinary tract infection
View Answer

a) Bloodstream infection

 

8. Which of the following is a symptom of meningitis that may develop over several days?
a) Seizures
b) Confusion
c) Sudden high fever
d) Stiff neck
View Answer

b) Confusion

 

9. What is the first-line treatment for bacterial meningitis?
a) Antiviral medication
b) Antibiotic treatment
c) Antifungal medication
d) Pain relievers
View Answer

b) Antibiotic treatment

 

10. Which division of the nervous system includes all neural elements except the brain and spinal cord?
a) Autonomic nervous system
b) Somatic nervous system
c) Central nervous system
d) Peripheral nervous system
View Answer

d) Peripheral nervous system

 

11. Which disorder involves the disruption of blood flow to the brain?
a) Epilepsy
b) Stroke
c) Meningitis
d) Neuralgia
View Answer

b) Stroke

 

12. What are the principal organs of the nervous system?
a) Brain and spinal cord
b) Heart and lungs
c) Liver and kidneys
d) Stomach and intestines
View Answer

a) Brain and spinal cord

 

13. Which type of infection involves inflammation of the brain tissue?
a) Meningitis
b) Encephalitis
c) Polio
d) Epidural abscess
View Answer

b) Encephalitis

 

14. Which disorder involves structural defects in the brain or spinal cord?
a) Meningitis
b) Epilepsy
c) Brain or spinal cord injury
d) Neuralgia
View Answer

c) Brain or spinal cord injury

 

15. Which symptom is NOT commonly associated with nervous system disorders?
a) Muscle rigidity
b) Loss of appetite or thirst
c) Weakness or loss of muscle strength
d) Nausea or vomiting
View Answer

d) Nausea or vomiting

 

16. What is the main symptom that distinguishes meningitis from the flu?
a) Muscle rigidity
b) Severe headache
c) Stiff neck
d) Loss of appetite or thirst
View Answer

c) Stiff neck

 

17. Which disorder involves degeneration of the nervous system?
a) Meningitis
b) Epilepsy
c) Peripheral neuropathy
d) Parkinson’s disease
View Answer

d) Parkinson’s disease

 

18. Which division of the nervous system controls involuntary functions such as heart rate and digestion?
a) Central nervous system
b) Somatic nervous system
c) Autonomic nervous system
d) Peripheral nervous system
View Answer

c) Autonomic nervous system

 

19. Which disorder involves abnormal electrical activity in the brain?
a) Meningitis
b) Stroke
c) Epilepsy
d) Neuralgia
View Answer

c) Epilepsy

 

20. What is the term for the sudden onset of a headache that seems different from normal?
a) Migraine
b) Cluster headache
c) Tension headache
d) Thunderclap headache
View Answer

d) Thunderclap headache

 

21. Which of the following is the most common cause of viral meningitis in the United States?
a) Influenza virus
b) Enteroviruses
c) Human papillomavirus
d) Varicella-zoster virus
View Answer

b) Enteroviruses

 

22. Which type of meningitis is typically contracted through breathing in fungal spores?
a) Viral meningitis
b) Bacterial meningitis
c) Fungal meningitis
d) Parasitic meningitis
View Answer

c) Fungal meningitis

 

23. Cryptococcal meningitis primarily affects individuals with:
a) Diabetes
b) Hypertension
c) Immune deficiencies
d) Allergies
View Answer

c) Immune deficiencies

 

24. How is parasitic meningitis typically transmitted to humans?
a) Through coughing and sneezing
b) Consuming contaminated food
c) Skin-to-skin contact
d) Sharing utensils
View Answer

b) Consuming contaminated food

 

25. Which of the following is NOT a recommended method for preventing meningitis?
a) Antibiotic treatment
b) Vaccination
c) Supportive therapy
d) Practice good hygiene
View Answer

a) Antibiotic treatment

 

26. Encephalitis is characterized by inflammation of the:
a) Spinal cord
b) Liver
c) Kidneys
d) Brain tissue
View Answer

d) Brain tissue

 

27. Which type of encephalitis occurs when a virus directly infects the brain and spinal cord?
a) Primary encephalitis
b) Secondary encephalitis
c) Tertiary encephalitis
d) Indirect encephalitis
View Answer

a) Primary encephalitis

 

28. What is the most common virus causing encephalitis in developed countries?
a) Influenza virus
b) Herpes simplex virus
c) Human papillomavirus
d) Hepatitis B virus
View Answer

b) Herpes simplex virus

 

29. Which part of the brain is typically affected by herpes encephalitis?
a) Occipital lobe
b) Parietal lobe
c) Temporal lobe
d) Cerebellum
View Answer

c) Temporal lobe

 

30. What diagnostic test is commonly performed to confirm encephalitis?
a) X-ray imaging
b) Urinalysis
c) Electrocardiogram (ECG)
d) Spinal tap or lumbar puncture
View Answer

d) Spinal tap or lumbar puncture

 

31. What is the mainstay of treatment for herpes encephalitis?
a) Antibiotics
b) Antifungal medications
c) Antiviral medications
d) Corticosteroids
View Answer

c) Antiviral medications

 

32. Traumatic brain injury specifically refers to a change in brain function caused by:
a) Genetic factors
b) Accidents or blunt force trauma
c) Infectious diseases
d) Chronic illnesses
View Answer

b) Accidents or blunt force trauma

 

33. How can traumatic brain injury affect a person?
a) Emotionally only
b) Cognitively only
c) Physically only
d) Cognitive, physical, emotional, or independent functioning
View Answer

d) Cognitive, physical, emotional, or independent functioning

 

34. Which of the following is NOT a symptom of mild encephalitis?
a) Fever
b) Vomiting
c) Hallucinations
d) Stiff neck
View Answer

c) Hallucinations

 

35. What distinguishes primary encephalitis from secondary encephalitis?
a) Primary encephalitis is caused by viruses, while secondary encephalitis is caused by bacteria.
b) Primary encephalitis directly infects the brain, while secondary encephalitis starts elsewhere in the body.
c) Primary encephalitis is more severe than secondary encephalitis.
d) Primary encephalitis is easier to treat than secondary encephalitis.
View Answer

b) Primary encephalitis directly infects the brain, while secondary encephalitis starts elsewhere in the body.

 

36. What is the primary cause of spinal cord injuries?
a) Infection
b) Genetic predisposition
c) Trauma
d) Aging
View Answer

c) Trauma

 

37. Which of the following is NOT a symptom of spinal cord injury?
a) Paralysis
b) Vomiting
c) Loss of consciousness
d) Fever
View Answer

d) Fever

 

38. What radiological test is considered the gold standard for assessing traumatic brain injuries?
a) MRI
b) X-ray
c) CT scan
d) PET scan
View Answer

c) CT scan

 

39. What type of injury results from compression of brain tissue underneath the cranium at the site of impact?
a) Contusion
b) Hematoma
c) Diffuse axonal injury
d) Skull fracture
View Answer

a) Contusion

 

40. Which of the following is NOT a potential cause of focal brain injury?
a) Sport injuries
b) Penetrating head wounds
c) Stroke
d) Congenital brain abnormalities
View Answer

d) Congenital brain abnormalities

 

41. What term is used to describe damage to the temporal pole resulting in speech difficulties?
a) Aphasia
b) Ataxia
c) Agnosia
d) Dysarthria
View Answer

a) Aphasia

 

42. Which type of brain injury is characterized by damage occurring at a specific location?
a) Focal injury
b) Diffuse injury
c) Axonal injury
d) Hemorrhagic injury
View Answer

a) Focal injury

 

43. What is the primary function of the Bobath Concept in neurological rehabilitation?
a) Sensory retraining
b) Strengthening and stretching
c) Balance reactions
d) Cognitive therapy
View Answer

c) Balance reactions

 

44. What secondary problem can result from brain injury-related swelling?
a) Hypoxia
b) Hyperactivity
c) Epilepsy
d) Vascular problems
View Answer

d) Vascular problems

 

45. Which symptom is NOT commonly associated with focal axonal brain injury?
a) Weakness
b) Emotional changes
c) Vision changes
d) Hearing loss
View Answer

d) Hearing loss

 

46. What is the term for injuries resulting from forces acting on the skull causing compression of brain tissue?
a) Contrecoup injuries
b) Acceleration injuries
c) Coup injuries
d) Diffuse injuries
View Answer

c) Coup injuries

 

47. Which type of brain injury is associated with acceleration/deceleration injuries?
a) Focal injury
b) Diffuse injury
c) Intracranial hemorrhage
d) Contusion
View Answer

b) Diffuse injury

 

48. Which test is NOT commonly used for radiological assessment of traumatic brain injuries?
a) CT scan
b) MRI
c) PET scan
d) X-ray
View Answer

c) PET scan

 

49. What is the term for a contained collection of bleeding in the brain?
a) Intracranial hemorrhage
b) Hematoma
c) Contusion
d) Diffuse axonal injury
View Answer

b) Hematoma

 

50. What symptom is associated with damage to the occipital pole of the brain?
a) Speech difficulties
b) Visual changes
c) Emotional changes
d) Weakness
View Answer

b) Visual changes

 

51. What is the primary goal of rehabilitation for survivors of traumatic brain injury?
a) Pain management
b) Maximizing recovery
c) Cognitive therapy
d) Preventing secondary swelling
View Answer

b) Maximizing recovery

 

52. Which type of injury results from forces causing compression of brain tissue opposite to the site of impact?
a) Diffuse axonal injury
b) Contrecoup injury
c) Hemorrhagic injury
d) Skull fracture
View Answer

b) Contrecoup injury

 

53. What type of injury is characterized by damage occurring over a widespread area of the brain?
a) Focal injury
b) Diffuse injury
c) Intracranial hemorrhage
d) Contusion
View Answer

b) Diffuse injury

54. Which of the following is NOT a symptom of focal brain injury?
a) Emotional changes
b) Difficulty swallowing
c) Balance difficulties
d) Dilated pupils
View Answer

d) Dilated pupils

 

55. What type of injury is associated with damage to vessels deep inside the skull?
a) Intracranial hemorrhage
b) Hematoma
c) Contusion
d) Diffuse axonal injury
View Answer

a) Intracranial hemorrhage

 

56. What type of brain injury results from tissue compression at the site of impact?
a) Contusion
b) Diffuse axonal injury
c) Intracerebral hemorrhage
d) Skull fracture
View Answer

a) Contusion

 

57. Which test is considered the gold standard for detecting the presence of blood and fractures in traumatic brain injury cases?
a) MRI
b) PET scan
c) X-ray
d) CT scan
View Answer

d) CT scan

 

58. What type of injury is associated with acceleration/deceleration forces on the brain tissue?
a) Focal injury
b) Diffuse injury
c) Intracranial hemorrhage
d) Contusion
View Answer

b) Diffuse injury

 

59. Which symptom is NOT commonly associated with traumatic brain injury?
a) Vomiting
b) Dizziness
c) Fever
d) Loss of consciousness
View Answer

c) Fever

 

60. What is the term for a problem with nerve, spinal cord, or brain function affecting a specific location?
a) Focal neurological deficit
b) Diffuse neurological deficit
c) Axonal neurological deficit
d) Hemorrhagic neurological deficit
View Answer

a) Focal neurological deficit

Download Link

Download PDF

Diseases Of Renal System

1. Which term is used interchangeably with nephritis?
a) Glomerulonephritis
b) Nephrosis
c) Renal failure
d) Cystitis
View Answer

a) Glomerulonephritis

 

2. What is the primary symptom of nephritis in its early stages?
a) Severe pain in the lower back
b) Foamy urine
c) Swelling in the extremities
d) Unexplained weight loss
View Answer

b) Foamy urine

 

3. What is a potential cause of nephritis?
a) Excessive vitamin intake
b) Genetic predisposition
c) Lack of physical activity
d) Overhydration
View Answer

b) Genetic predisposition

 

4. How is nephritis typically diagnosed?
a) X-ray imaging
b) Biopsy
c) MRI scan
d) Blood pressure monitoring
View Answer

b) Biopsy

 

5. Which of the following is NOT a symptom of nephrosis?
a) Foamy urine
b) Weight gain
c) Severe swelling
d) Increased appetite
View Answer

d) Increased appetite

 

6. What is a common cause of nephrosis?
a) Hypertension
b) Systemic lupus erythematosus
c) Dehydration
d) Vitamin deficiency
View Answer

b) Systemic lupus erythematosus

 

7. How is nephrotic syndrome typically detected?
a) Blood tests
b) X-ray imaging
c) Echocardiogram
d) Electroencephalogram (EEG)
View Answer

a) Blood tests

 

8. What is one of the earliest signs of kidney disease?
a) Protein in the urine
b) High blood pressure
c) Elevated cholesterol levels
d) Abnormal kidney size
View Answer

a) Protein in the urine

 

9. What diagnostic procedure involves examining a small piece of kidney tissue under a microscope?
a) CT scan
b) Ultrasound
c) Biopsy
d) Urinalysis
View Answer

c) Biopsy

 

10. Which medication is commonly prescribed to reduce protein release in urine for nephrosis patients?
a) Antibiotics
b) Anticoagulants
c) ACE inhibitors
d) Antidepressants
View Answer

c) ACE inhibitors

 

11. What is the function of diuretics in nephrosis treatment?
a) Increase blood pressure
b) Reduce cholesterol levels
c) Promote fluid retention
d) Decrease excess fluid buildup
View Answer

d) Decrease excess fluid buildup

 

12. Which condition can nephrosis be a complication of?
a) Hypothyroidism
b) Diabetes
c) Asthma
d) Osteoporosis
View Answer

b) Diabetes

 

13. Which of the following is NOT a treatment option for nephrosis?
a) Immune system-suppressing medications
b) Blood thinners
c) Antibiotics
d) Cholesterol-reducing medications
View Answer

c) Antibiotics

 

14. What role do blood thinners play in nephrosis treatment?
a) Reduce protein in urine
b) Control blood pressure
c) Prevent blood clots
d) Improve kidney function
View Answer

c) Prevent blood clots

 

15. Which disease can lead to diabetic nephropathy?
a) Hyperthyroidism
b) Osteoarthritis
c) Systemic lupus erythematosus
d) Diabetes
View Answer

d) Diabetes

 

16. What do urine tests for nephrotic syndrome primarily detect?
a) Blood sugar levels
b) Presence of bacteria
c) Protein in urine
d) pH levels
View Answer

c) Protein in urine

 

17. What is the primary function of cholesterol-reducing medications in nephrosis treatment?
a) Reduce proteinuria
b) Lower blood pressure
c) Improve cholesterol levels
d) Stimulate kidney function
View Answer

c) Improve cholesterol levels

 

18. Which medication category is used to suppress the immune system in nephrosis treatment?
a) Antibiotics
b) Diuretics
c) Immune system-suppressing medications
d) Analgesics
View Answer

c) Immune system-suppressing medications

 

19. What is the main aim of blood pressure medications in nephrosis treatment?
a) Increase urine production
b) Reduce fluid retention
c) Promote blood clotting
d) Lower blood pressure
View Answer

d) Lower blood pressure

 

20. Which term is used to describe any degenerative disease of the kidney tubules?
a) Glomerulonephritis
b) Nephrosis
c) Diabetic nephropathy
d) Focal segmental glomerulosclerosis
View Answer

b) Nephrosis

 

21. What is the primary cause of acute pyelonephritis?
a) Gram-positive bacteria
b) Viruses
c) Gram-negative bacteria
d) Fungal infection
View Answer

c) Gram-negative bacteria

 

22. Which of the following symptoms is NOT typically associated with cystitis?
a) Dark, cloudy urine
b) Pain in the back
c) Burning sensation while urinating
d) Feeling generally unwell
View Answer

b) Pain in the back

 

23. What imaging technique involves injecting a radioactive material to visualize infected or scarred areas in the kidneys?
a) MRI
b) Ultrasound
c) Radioactive imaging
d) X-ray
View Answer

c) Radioactive imaging

 

24. Which antibiotic is NOT listed as a treatment option for acute pyelonephritis?
a) Levofloxacin
b) Ciprofloxacin
c) Co-trimoxazole
d) Penicillin
View Answer

d) Penicillin

 

25. What is the term used to describe stones formed in the urinary tract?
a) Urethritis
b) Urolithiasis
c) Cystolithiasis
d) Nephrolithiasis
View Answer

b) Urolithiasis

 

26. Which of the following is NOT a symptom of pyelonephritis?
a) Nausea
b) Cloudy urine
c) Flank tenderness
d) Pink urine
View Answer

d) Pink urine

 

27. What is the primary cause of cystitis in most cases?
a) Overhydration
b) Bacterial infection
c) Viral infection
d) Genetic predisposition
View Answer

b) Bacterial infection

 

28. Which imaging test involves a thin tube with a camera and light to inspect the bladder?
a) X-ray
b) Ultrasound
c) MRI
d) Cystoscopy
View Answer

d) Cystoscopy

 

29. What might be a possible complication of pyelonephritis?
a) Hypertension
b) Appendicitis
c) Kidney failure
d) Bone fracture
View Answer

c) Kidney failure

 

30. Which of the following is NOT a symptom of cystitis?
a) Pain during urination
b) Dark, cloudy urine
c) Pain low down in the tummy
d) Increased appetite
View Answer

d) Increased appetite

 

31. What is the most common bacterium causing acute pyelonephritis?
a) Staphylococcus aureus
b) Escherichia coli
c) Streptococcus pneumoniae
d) Clostridium difficile
View Answer

b) Escherichia coli

 

32. What medication is commonly used to treat bacterial cystitis?
a) Antihistamines
b) Antifungals
c) Antibiotics
d) Antivirals
View Answer

c) Antibiotics

 

33. Which symptom is not commonly associated with urolithiasis?
a) Nausea and vomiting
b) Pink, red or brown urine
c) Cloudy or foul-smelling urine
d) Itchiness in the groin area
View Answer

d) Itchiness in the groin area

 

34. What might be a cause of cystitis other than a urinary tract infection?
a) Excessive water intake
b) Overuse of antibiotics
c) Certain medications or hygiene products
d) Lack of physical exercise
View Answer

c) Certain medications or hygiene products

 

35. Which of the following is NOT a method for diagnosing cystitis?
a) Urine tests
b) Blood tests
c) Cystoscopy
d) Imaging tests
View Answer

b) Blood tests

 

36. What mineral deposits are typically found in kidney stones?
a) Calcium and phosphate
b) Iron and magnesium
c) Sodium and potassium
d) Zinc and copper
View Answer

a) Calcium and phosphate

 

37. What is the primary treatment for acute pyelonephritis?
a) Surgery
b) Painkillers
c) Antibiotics
d) Physical therapy
View Answer

c) Antibiotics

 

38. Which of the following is NOT a potential cause of urolithiasis?
a) Dehydration
b) Genetics
c) Certain medications
d) Overconsumption of dairy products
View Answer

d) Overconsumption of dairy products

 

39. What might be a symptom unique to interstitial cystitis?
a) Dark urine
b) Urgent need to urinate
c) Pain during urination
d) Pain relieved by urination
View Answer

d) Pain relieved by urination

 

40. What imaging technique involves using sound waves to create pictures of the urinary tract?
a) X-ray
b) MRI
c) Ultrasound
d) Radioactive imaging
View Answer

c) Ultrasound

 

41. Which imaging modality is typically used for the diagnosis of nephrolithiasis?
a) MRI
b) Ultrasound
c) Noncontrast helical CT
d) X-ray
View Answer

c) Noncontrast helical CT

 

42. What is the primary treatment for kidney stones involving shock waves?
a) Ureteroscopy
b) Percutaneous nephrolithotomy
c) Shock wave lithotripsy
d) Medical expulsive therapy
View Answer

c) Shock wave lithotripsy

 

43. Which procedure involves the use of a long tube-like tool to find and remove kidney stones?
a) Shock wave lithotripsy
b) Ureteroscopy
c) Percutaneous nephrolithotomy
d) Urinalysis
View Answer

b) Ureteroscopy

 

44. What is the purpose of using a laser during ureteroscopy?
a) To induce shock waves
b) To sterilize the urinary tract
c) To break the kidney stone into small pieces
d) To analyze the composition of the stone
View Answer

c) To break the kidney stone into small pieces

 

45. Which surgery involves the insertion of a tube directly into the kidney to remove stones?
a) Shock wave lithotripsy
b) Ureteroscopy
c) Percutaneous nephrolithotomy
d) Cystoscopy
View Answer

c) Percutaneous nephrolithotomy

 

46. In which case would shock wave lithotripsy NOT be the preferred treatment option?
a) Small kidney stones
b) Large kidney stones
c) Stones causing infection
d) Stones resistant to laser therapy
View Answer

b) Large kidney stones

 

47. Which diagnostic method is typically used to detect stones in the urinary tract without contrast?
a) Magnetic resonance imaging (MRI)
b) Noncontrast helical CT
c) Ultrasound
d) Intravenous pyelogram (IVP)
View Answer

c) Ultrasound

 

48. What is the duration of hospitalization typically associated with percutaneous nephrolithotomy?
a) 1 day
b) 2-3 days
c) 5-7 days
d) Outpatient procedure
View Answer

b) 2-3 days

 

49. Which condition is NOT listed as a potential cause of nephrolithiasis?
a) Cystic kidney disease
b) Swelling in the joints
c) High levels of uric acid in urine
d) Intestinal surgery
View Answer

b) Swelling in the joints

 

50. What is the primary purpose of medical expulsive therapy in the treatment of kidney stones?
a) To prevent recurrence of stones
b) To reduce pain and discomfort
c) To dissolve the stones chemically
d) To facilitate the passage of stones through the urinary tract
View Answer

d) To facilitate the passage of stones through the urinary tract

Download Link

Download PDF

Download the PDF files containing the MCQs sets using the provided links above. Allocate a suitable time to attempt each set of questions. Answer each question to the best of your knowledge. After completing each set, review your answers and identify areas for improvement. Utilize these MCQs as a tool for reinforcing your understanding of Veterinary Medicine MCQs With Answers PDF Free Download.

You might also be interested in:

Veterinary Pathology MCQs With Answers PDF Free Download [Direct Link]

Veterinary Pathology MCQs With Answers PDF

Disclaimer

disclaimer

This blog post is just for education purpose and to convey important information to our audience. The content that we are publishing on our website is not collected or plagiarized from any source.. We are just sharing our experience If anyone finds a text that is plagiarized from any source in our articles, please send us an email at Rizqum789@gmail.com and we will remove it within 24 hours.

The Vets Republic website has initiated its journey to convey information that is useful for the veterinarian audience. Anything that we have published here is just for educational purposes. We have not posted any copyrighted content Your continuous support would be highly appreciated.

 

vetsrepublic.com
vetsrepublic.comhttp://vetsrepublic.com
Welcome to the wild world of veterinary magic! I'm Dr. Rizwan Qambrani, an intrepid explorer pursuing my Doctor of Veterinary Medicine (DVM) degree at the illustrious Lasbela University of Agriculture, Water, and Marine Sciences (Balochistan, Pakistan). With boundless enthusiasm and a heart full of love for all creatures, I'm on a thrilling mission to unravel the mysteries of animal health and well-being. Join me on this exhilarating journey as we embark on daring rescues, heartwarming tales, and a symphony of wagging tails and happy meows! Let's dive headfirst into the paw-some world of veterinary adventure! :)
RELATED ARTICLES

LEAVE A REPLY

Please enter your comment!
Please enter your name here

Most Popular

Recent Comments